Last visit was: 26 Jul 2024, 20:47 It is currently 26 Jul 2024, 20:47
Close
GMAT Club Daily Prep
Thank you for using the timer - this advanced tool can estimate your performance and suggest more practice questions. We have subscribed you to Daily Prep Questions via email.

Customized
for You

we will pick new questions that match your level based on your Timer History

Track
Your Progress

every week, we’ll send you an estimated GMAT score based on your performance

Practice
Pays

we will pick new questions that match your level based on your Timer History
Not interested in getting valuable practice questions and articles delivered to your email? No problem, unsubscribe here.
Close
Request Expert Reply
Confirm Cancel
SORT BY:
Date
   1   2   3   4  ...  16   
Magoosh GMAT Instructor
Joined: 28 Dec 2011
Posts: 4490
Own Kudos [?]: 28872 [0]
Given Kudos: 130
Magoosh GMAT Instructor
Joined: 28 Dec 2011
Posts: 4490
Own Kudos [?]: 28872 [0]
Given Kudos: 130
Magoosh GMAT Instructor
Joined: 28 Dec 2011
Posts: 4490
Own Kudos [?]: 28872 [0]
Given Kudos: 130
Magoosh GMAT Instructor
Joined: 28 Dec 2011
Posts: 4490
Own Kudos [?]: 28872 [0]
Given Kudos: 130
4 Challenging Comparison Questions on the GMAT [#permalink]
Expert Reply
FROM Magoosh Blog: 4 Challenging Comparison Questions on the GMAT
First of all, here are four challenging SC questions involving comparisons.  What could be better than SC questions about comparisons? (I couldn’t resist starting off a blog about comparisons with a comparison!)

 

1) For parts of his life, Burroughs, a writer in self-imposed exile, and in a similar way, Joyce lived a generation earlier, leaving Ireland and choosing to write about his homeland from a distance.

(A) Burroughs, a writer in self-imposed exile, and in a similar way, Joyce lived a generation earlier, leaving Ireland and choosing

(B) Burroughs was a writer in self-imposed exile, close to the way that Joyce lived a generation earlier, leaving Ireland and choosing

(C) Burroughs was a writer in self-imposed exile, somewhat as Joyce, who lived a generation earlier, left Ireland and chose

(D) Burroughs was a writer in self-imposed exile; a generation earlier, Joyce was a similar writer in self-imposed exile, leaving Ireland to choose

(E) a writer in self-imposed exile, Burroughs was similar to Joyce a generation earlier, leaving Ireland and choosing

 

2) Many acids corrode many metals, such as iron and copper, and oxygen does the same thing to them.

(A) Many acids corrode many metals, such as iron and copper, and oxygen itself does the same thing to them.

(B) Like many acids, oxygen itself has a corrosive effect on many metals, such as iron and copper

(C) Oxygen is like many acids in its ability to generate corrosive effect on many metals, such as and copper.

(D) Many metals, such as iron and copper, are corroded by oxygen itself; similarly, they are corroded by many acids.

(E) Corroding many metals, such as iron and copper, is a property of many acids, and, like these acids, of oxygen itself.

 

3) Zhuangzi had an interpretation of Daoism that was highly imaginative, but in terms of a lasting impact on the course of Chinese civilization this was not as influential as the interpretation of Confucianism of Mengzi, who was his contemporary.

(A) Zhuangzi had an interpretation of Daoism that was highly imaginative, but in terms of a lasting impact on the course of Chinese civilization this was not as influential as the interpretation of Confucianism of Mengzi, who was his contemporary

(B) Mengzi’s interpretation of Confucianism had a lasting impact on the course of Chinese civilization, whereas his contemporary Zhuangzi’s interpretation of Daoism, though highly imaginative, did not do this

(C) Zhuangzi had a highly imaginative interpretation of Daoism, but this interpretation had less of an impact on the course of Chinese civilization than his contemporary Mengzi, whose interpretation of Confucianism was more influential

(D) Zhuangzi’s interpretation of Daoism, though highly imaginative, did not have as lasting an impact on the course of Chinese civilization as had his contemporary Mengzi’s interpretation of Confucianism

(E) In terms of lasting impact, Mengzi’s interpretation of Confucianism influenced Chinese civilization more than the highly imaginative interpretation of Daoism by Mengzi’s contemporary Zhuangzi

 

4) A work with merits, The History of the United States (1801-1817), which Henry Adams thought was his masterpiece, has been neglected, and therefore, ironically, it is not as popular as his memoirs, The Education of Henry Adams, which he intended as a small publication for friends and which, after his death, went on to win numerous accolades, such as the Pulitzer Prize.

(A) A work with merits, The History of the United States (1801-1817), which Henry Adams thought was his masterpiece, has been neglected, and therefore, ironically, it is not as popular as his memoirs, The Education of Henry Adams, which he merely intended as a private publication for friends and which, after his death, went on to win numerous accolades, such as the Pulitzer Prize

(B) Henry Adams intended that his memoirs, The Education of Henry Adams, would be merely a private publication for friends, but it became a classic after his death, winning numerous accolades including a Pulitzer Prize; however, his ironic masterpiece, The History of the United States (1801-1817), was neglected

(C) Ultimately much more popular, winning numerous accolades including a Pulitzer Prize, the classic The Education of Henry Adams, the memoir published by Henry Adams and intended merely as a private publication for friends, as ironically compared to his neglected masterpiece, The History of the United States (1801-1817)

(D) Winning numerous accolades including a Pulitzer Prize, the classic The Education of Henry Adams, which he intended as a mere private publication for friends, was much more popular after his death then his masterpiece The History of the United States (1801-1817), which despite its merits was neglected with irony

(E) Henry Adams thought that his The History of the United States (1801-1817) was his masterpiece, but despite its many merits, this work has been neglected; ironically, his memoirs, The Education of Henry Adams, intended merely as a private publication for friends, became a classic after his death, winning numerous accolades including a Pulitzer Prize

 

Explanations for these problems will come at the end of this article.

 

Comparisons on the GMAT
Here is a list of other blog articles that discuss issues relevant to comparisons on the GMAT Sentence Correction.  I include in this list articles about parallelism, because comparisons are a special case of parallelism, and everything that is true about parallelism applies to comparison as well.  (Notice the complex comparison in the previous sentence!)

a) GMAT Sentence Correction: Comparisons

b) GMAT Sentence Correction: Parallelism

c) Parallelism on the GMAT Sentence Correction

d) GMAT Parallelism: Once Outside or Twice Inside

e) Dropping Common Words in Parallel on the GMAT

f) GMAT Sentence Correction: like vs. as

g) GMAT Comparisons: More vs. Greater and Less vs. Fewer

h) GMAT Idioms of Comparisons

i) GMAT Tuesdays with Kevin: Sentence Correction—Spotting Parallelism

j) GMAT Tuesdays with Kevin: Sentence Correction—Comparisons and Ellipses

Those articles cover many of the basic rules.  One way to practice is to look at all the comparisons all around us.  What comparisons are typical of advertisements?  What comparisons are typical of PR folks speaking for corporations?  What comparisons are typically for politicians to make?  Get curious about the which kinds of comparisons are more typical in which niches in the real world.  (Notice that the last sentence involved a comparison with relative clauses.)

 

One typical logical problem in comparisons
One of the GMAT’s favorite mistakes in comparison questions involves the confusion between subjective comparisons and objective comparisons.  If the main verb is intransitive, that is, a verb that does not take a direct object, then this ambiguity does not arise.

5) Chris runs faster than Mike, but Mike walks as fast as Chris.

In either half of that sentence, there is no ambiguity.  Clearly, in both cases, we are comparing Mike and Chris.

When the main verb is transitive, that is, one that takes a direct object, and the comparative term follows the [subject][verb][object] structure, then it can be unclear whether the intended comparison is with the subject or with the object, i.e. a subjective comparison vs. an objective comparison.  Consider this faulty sentence.

6a) Mike likes opera more than Chris.

That is a logically flawed sentenced, because it is open to two different readings:

6b) (the subjective comparison) Mike likes opera more than Chris does.

6c) (the objective comparison) Mike likes opera more than he likes Chris.

Version (6a) is logically flawed, because it does not allow us to distinguish between these two readings, the subjective comparison vs. the objective comparison.  Obviously, this is a very simply example sentence.  Do you spot the answer choices in the practice questions above in which this mistake is repeated?

FWIW, while I tremendously enjoy many operas, including Don Giovanni, Carmen, and Tristan und Isolde, I have considerably greater affection and esteem for my brilliant friend Chris, who, as it happens, is not nearly as fond of opera as I am. (That’s a very sophisticated comparison!)  De gustibus non est disputandum.

 

More sophisticated comparisons
For many of the more sophisticated SC questions involving comparisons, learning the rules of comparisons is not enough.  Beyond good grammar, a sophisticated sentence also employs superb logic and rhetoric.  Logical predication requires that comparisons compare like to like, and every comparison is completely clear and unambiguous.  Among other things, it involves not ambiguity between subjective comparisons and objective comparison: such ambiguity is a typical GMAT SC comparison mistake pattern.

Rhetorical construction is, in some ways, the most challenging aspect of GMAT Sentence Correction, because is the least rule-based and the most dependent on an “ear” for the language.   Non-native speakers find it particularly challenging for this reason.   Certainly part of rhetoric is concision, and part is avoiding redundancy.   Most of it, though, concerns the holistic structure of a sentence: is the sentence organized in a way that is powerfully cogent and meaningful?  In the answer choices of the four sentences above, the grammar rules of comparisons are obeyed in most of them, but there are a smattering of logic problems, and different answer choices are more or less successful rhetorically.

How does one learn what “rhetorically successful” means, especially if one is a non-native speaker?  This answer is simple to say: develop a habit of reading.   Someone aspiring to GMAT Sentence Correction mastery has to force herself to push through the most challenging and sophisticated reading possible on a daily basis.  It only through practice and repeated exposure that the brain can pick up the subtle patterns, and such acquisition leads to mastery.  I would also strongly urge you to read exceedingly carefully all the official explanations of rhetorically challenging Sentence Correction questions, such as those here; remember, you are looking, not so much for rules as for patterns.  Rhetoric is more about patterns than about rules.  (Both of those last sentences had wonderful comparisons!)

 

Summary
If the foregoing discussion gave you some insights, you may give the questions above another look.  If the four practice questions here still puzzle you, then read the official solutions below very carefully.  Continue to pay attention to comparisons in the real world.  Continue to push yourself through sophisticated reading.  To get a score higher than most people’s scores, you have to put in a more greater than effort than most people do.  (With that final comparison, we will conclude!)



 

Practice problem explanations
1) (A) This is a sloppy illogical comparison.  It sounds as if Joyce, in living a generation earlier, was thereby similar to Burroughs.  The fact that Joyce was an expat writer seems incidental and unrelated to the comparison.  We gather that we are trying to compare two expat writers, and this version does not do this successfully.  Choice (A) is incorrect.

(B) This comparison is also illogical.  Burroughs was a writer the way that Joyce lived a generation earlier: one man’s writing is like the other man’s living.  This is illogical.  Choice (B) is incorrect.

(C) This is logically and grammatically correct.  It compares Burroughs as an expat writer to Joyce as an expat writer.  This is clear, direct, and strong.  This choice is promising.

(D) This changes the meaning.  What is similar about the two men is that they were both expat writers; the original does not imply that they had similar writing styles, but this is precisely what (D) suggests, calling Joyce a “similar writer.”  Joyce did a similar thing, but he didn’t write in a similar way.  Choice (D) is incorrect.

(E) This version makes it sound as if Burroughs, like Joyce, left Ireland.  That is not part of the meaning of the original sentence.

The only possible answer is (C).

 

2) (A) the prompt is somewhat unclear in meaning, and “does the same thing to them” is a very colloquial construction that would not be considered acceptable on the GMAT.  Choice (A) is incorrect.

(B) Here, the comparison is very clear.  This is grammatical correct and logically sound, a promising choice.

(C) The phrase “in its ability to generate corrosive effects” seems to be in a contest for longest and most awkward wording.  This is a ridiculously long way to say “can corrode.”  Choice (C) is incorrect.

(D) The passive structure is less than desirable.  Also, there’s a subtle rhetorical problem here.  The prompt and other choices suggest that it is expected that acids corrode metals, but it is a surprising departure from the norm that oxygen does as well.  Rhetorically, it is unusual to lead with the exception and then, as an afterthought, add the rule.  Choice (D) is incorrect.

(E) This is a very awkward indirect way of conveying the information.  The main action is congealed into a gerund and made the subject of the sentence, which has the effect of making the entire sentence less active and forceful.  Choice (E) is incorrect.

The only possible answer is (B).

 

3) A question about the great Daoist thinker Zhuangzi and the great Confucian thinker Mengzi, known in the West as “Mencius.”

(A) This option is grammatically correct, but it is indirect and wordy: the phrase “in terms of a lasting impact on the course of Chinese civilization this was not as influential as” is a bloated disaster.  Also, the double “of” prepositional phrases are awkward and ambiguous: “the interpretation of Confucianism of Mengzi.” This option is not correct.

(B) This option looks promising right up to the final word.  The pronoun “this” has as its antecedent an action, and a pronoun can’t represent an action.  The proper way to refer to a previously stated predicate is “to do so.”  If this option had ended with “did not do so,” then this would have been a compelling choice for the answer.  As it stands, this is incorrect.

(C) This has a few problems.  First of all, the repetition of the long word “interpretation” is a little clumsy.  More importantly, this commits the logical mistake discussed above.  Let

Z = Zhuangzi’s imaginative interpretation

CC = course of Chinese civilization

M = Mengzi

This sentence states

“Z had less impact on CC than M”

Is this a subjective or objective comparison??

subjective: “Z had less impact on CC than did M”

objective: “Z had less impact on CC than it had on M”

We get logically that the subjective comparison is the intended one, but the grammar does not make that clear: the wording as it stands is ambiguity, so this cannot possible be a correct answer choice.

(D) This one is grammatically correct, logically clear, and rhetorically sound.  This is a promising choice.

(E) Same logic mistake.  Let

M = Mengzi’s interpretation of Confucianism

CC = Chinese civilization

Z = the highly imaginative interpretation of Daoism by Mengzi’s contemporary Zhuangzi

This sentence states:

“M influenced CC more than Z”

Is this a subjective or objective comparison??

subjective: “M influenced CC more than Z did”

objective: “M influenced CC more than it influenced Z”

Again, we get logically that the subjective comparison is the intended one, but the grammar does not make that clear: the wording as it stands is ambiguity, so this cannot possible be a correct answer choice.

The only possible answer is (D).

 

4) A sentence about the great scholar Henry Adams (1838 – 1918)

Choice (A) is grammatically correct but clunky.  Notice that a lot of this sentence is passive: the books are the subjects, receiving the action of Henry Adams.  This makes a long sentence even wordier.  Rhetorically, this is a very poor choice.  Choice (A) is not out-and-out incorrect, but we dearly hope that we can find something better!

In Choice (B), the “irony” lies in the relative reception of the two books, despite Adam’s intentions.  It is incorrect to say that the masterpiece itself was “ironic.”  Also, notice there’s a subtle shift in meaning from saying that Adams “thought” that the History book was a masterpiece to simply calling it a masterpiece.   Choice (B) is incorrect.

In Choice (C), the phrase “as ironically compared” is very awkward.  The comparing itself is not necessarily done ironically.   The phrase after this is compressed, and doesn’t convey the entire ironic relationship of the two books.  Choice (C) is incorrect.

In Choice (D), the phrase “neglected with irony” suggests that the act of neglecting itself was done with irony.  That meaning differs from the prompt.  Also, it’s a little odd that the man’s name, Henry Adams, is introduced only via the title of his memoirs, and this mention in the title serves as the antecedent of a pronoun.  Choice (D) is incorrect.

Choice (E) has two active independent clauses, one on each side of the semicolon.  The semicolon allows us to “take a breath”: the pause helps to organize the information in a logical way.  The appearance of the word “ironically” immediately after the semicolon correctly situates the irony in the relationship of the two books and their relative receptions.  This is a considerably better answer than (A), and it is the best answer here.

Answer = (E)

 

The post 4 Challenging Comparison Questions on the GMAT appeared first on Magoosh GMAT Blog.
This Blog post was imported into the forum automatically. We hope you found it helpful. Please use the Kudos button if you did, or please PM/DM me if you found it disruptive and I will take care of it. -BB
Magoosh GMAT Instructor
Joined: 28 Dec 2011
Posts: 4490
Own Kudos [?]: 28872 [0]
Given Kudos: 130
GMAT Tuesday: OG Reading Comprehension #101 [#permalink]
Expert Reply
FROM Magoosh Blog: GMAT Tuesday: OG Reading Comprehension #101
This week we tackle another inference question from the Official Guide for GMAT. In a lot of ways it builds nicely on the previous question since we are making an inference about the same group of patients from question number 100.



 

If you’d like to see question 99, you can find that video by clicking here.

 

Then, check out this week’s board:


 

 

The post GMAT Tuesday: OG Reading Comprehension #101 appeared first on Magoosh GMAT Blog.
This Blog post was imported into the forum automatically. We hope you found it helpful. Please use the Kudos button if you did, or please PM/DM me if you found it disruptive and I will take care of it. -BB
Magoosh GMAT Instructor
Joined: 28 Dec 2011
Posts: 4490
Own Kudos [?]: 28872 [0]
Given Kudos: 130
How to Choose Which Business School Offer to Accept [#permalink]
Expert Reply
FROM Magoosh Blog: How to Choose Which Business School Offer to Accept
Getting into more than one business school is never a bad thing, but it does leave you

with a tough decision. Unless you’ve had your sights set on a particular MBA program, you’re likely to find yourself torn when choosing between your business school offers, wondering how this monumental choice will affect your future. Here are a few tips to help you evaluate which of your offers to accept.

 

Consider the Little Things
You’ve probably already done the obvious and weighed the schools’ rankings against one another, but just because a particular MBA program is considered the “best” by some, does not mean it is best for you. There is more to your MBA experience than numbers alone.

For instance, the geographical location of the program can play an important role in your business school career. If you favor a small-town environment, a school in a big city might prove overwhelming or anxiety inducing. If you’re more of an urban person, on the other hand, you might find yourself growing restless in a slower-paced atmosphere. On a similar note, you’ll want to take into account the size of the student body and ask yourself whether you’re more likely to thrive in a large or small student population.

Equally important is to weigh where the employers are and the potential benefits of attending a school in their own backyard.

 

Get Personal
Your decision will be all the more difficult if you’re comparing schools that you’ve only met on paper. You can research rankings and alumni salaries to your heart’s content, but sometimes, the decision boils down to a gut feeling. If you haven’t already, take a trip to each school that you’re considering to give yourself a closer look. Where did you feel most comfortable? Which atmosphere struck you as more conducive to your success? Whose students did you feel more drawn to?

In addition to seeing the school in person, you should also try to contact alumni – especially those with positions in your target industry – to learn about their experiences during and after the program and assess how open they are to forming connections with you. Business school offers you only half of what you need to succeed; the other half comes from the networks you build while you’re there.

 

Do the Math
And then, of course, there are those good, old quantifiable data points, though there are several factors to consider beyond the schools’ rankings. Is there a significant difference in tuition costs? Are one program’s students more likely to find successful careers in your desired industry? What percentage of your dream company’s employees studied at each school?

 

Congratulate Yourself
Remember, if you’re stuck in this particular predicament, it means you’ve impressed quite a few people who are notoriously difficult to impress. You applied to each of these programs because you considered them suited to your goals. Pat yourself on the back, take a deep breath and know that you have the potential to succeed in whichever school you select. It’s all in the details now.

 

 

The post How to Choose Which Business School Offer to Accept appeared first on Magoosh GMAT Blog.
This Blog post was imported into the forum automatically. We hope you found it helpful. Please use the Kudos button if you did, or please PM/DM me if you found it disruptive and I will take care of it. -BB
Magoosh GMAT Instructor
Joined: 28 Dec 2011
Posts: 4490
Own Kudos [?]: 28872 [0]
Given Kudos: 130
5 Essential GMAT Study Tips [#permalink]
Expert Reply
FROM Magoosh Blog: 5 Essential GMAT Study Tips
If you’re headed for an MBA, the first thing you’ll need to do is beat the GMAT. This is a challenging hoop to jump through. But the GMAT can be beat. And studying well for the GMAT will not only help you get a top score—it’ll also help you build valuable skills for your future business studies. In this post, we’ll look at five essential GMAT study tips that can help you on test day and beyond.

 

Study Tip 1: Use Official Prep Materials
Test prep materials from the makers of the exam—the Graduate Management Admissions Council—are absolutely essential to successful GMAT Prep. These official materials are must-haves for your GMAT prep because the exam has a highly specific, often updated format. No other third-party source of GMAT prep is guaranteed to capture the feel of the real exam as completely as materials form the actual makers of the test

GMAC offers official materials through its test prep website, MBA.com. Official prep resources include GMAT’s free question sets for Verbal, Integrated Reasoning, Quantitative, and it’s official AWA practice web page. You’ll also want to purchase the latest edition of the GMAT official guides, and other official prep materials offered by GMAC.

 

Study Tip 2: Choose unofficial prep materials by comparing them to the real thing
The more study resources you have for your GMAT preparation, the better. So you’ll definitely want to use some third-party GMAT content. But be a smart consumer. There are many GMAT prep materials out there, offered through numerous publishers and websites. Use only non-GMAC materials that measure up well to the real thing. If you’re having trouble determining whether an outside resource is true to the exam itself, check out reviews of pep books and websites (like the ones we have here on Magoosh), and consult forums for GMAT preppers, such as GMAT Club and the Beat the GMAT Forums.

 

Study Tip 3: Watch the clock
Pacing is very important on the GMAT. When you sit for the test, you’ll be given many rigorous, demanding multistep academic tasks and a limited amount of time to do them. Learning the content of the exam is very important, but it’s only half the battle. If you develop the math, verbal, writing, and reasoning skills you need on the test but don’t learn how to use these skills quickly, you’ll still find yourself missing a lot of questions for lack of time. So include plenty of pacing practice as part of your overall test prep.

 

Study Tip 4: Train yourself to do things in your head
As Magoosh GMAT expert Mike McGarry will tell you, mental math is very important to pacing and accuracy on the Quantitative and Integrated Reasoning portions of the exam. And really, this kind of “mindwork” is really important as you practice for every section of the exam. For Verbal, learn to read with minimal note-taking— note-taking is important, but overly heavy note-taking can slow you down and end even distract you. And for AWA, focus on ways to simplify your pre-writing. The more you can hold your essay in your mind, the quicker you’ll be able to start typing your AWA essay into the exam computer. Do not rely on heavy notes and elaborate outlines as you practice for the AWA.

 

Study Tip 5: Practice visual literacy
The ability to read symbols, graphs, tables and charts is called “visual literacy.” And this kind of reading ability is very important to the GMAT. In GMAT Quants, you’ll be expected to read geometric figures, coordinate plane plots and other math graphics with ease. And you’ll need to be comfortable using both established notational symbols for advanced math and strange symbols created just for the exam.

Also remember that GMAT Integrated Reasoning requires an even higher level of visual literacy than Math. In the IR section, you’ll need to simultaneously look through multiple complex charts and tables to gather data.

So be sure to really practice visual literacy skills during your GMAT prep. Time yourself to see how quickly you can read, interpret and extract answers from visuals in GMAT Quants and Integrated Reasoning. And try to have fun with visuals too—peruse infographic websites like Information is Beautiful.

Take some time to really play with visual literacy too—the GMAT makes up its own imaginary math symbols, and you should too. Think of a particular math function that doesn’t have a symbol and invent one for it, applying the new symbol to practice equations that you come up with on our own. If you’re feeling really ambitious, you can even make your own coordinate plane graphs and geometric figures.

 

The post 5 Essential GMAT Study Tips appeared first on Magoosh GMAT Blog.
This Blog post was imported into the forum automatically. We hope you found it helpful. Please use the Kudos button if you did, or please PM/DM me if you found it disruptive and I will take care of it. -BB
Magoosh GMAT Instructor
Joined: 28 Dec 2011
Posts: 4490
Own Kudos [?]: 28872 [0]
Given Kudos: 130
GMAT Critical Reasoning: More on Assumption Questions [#permalink]
Expert Reply
FROM Magoosh Blog: GMAT Critical Reasoning: More on Assumption Questions
First, four practice CR questions asking for the assumptions of arguments.

1) In anticipation of the coming year, Tecumseh Autos, a national auto manufacturer, is anticipating sales of its vehicles.  Tecumseh manufactures compact cars, sedans, minivans, trucks, SUVs, and sports cars.  In all categories of vehicles, Tecumseh sets prices so that the profit per vehicle is, on average, about the same.   Since the best indicator for sales in each category are the sales last year, Tecumseh’s marketing analysts’ prediction of the three most profitable categories of vehicles in the coming year will be compact cars, minivans, and SUVs respectively.

Tecumseh’s marketing analysts’ prediction relies on which one of the following assumptions?

(A) Across all manufacturers, the most popular cars on the road in America are compact cars, minivans, and SUVs.

(B) The models of Tecumseh’s compact cars to be sold in the upcoming year are identical to or similar to those of last year.

(C) Last year, no other category of Tecumseh’s vehicles generated more profits than SUVs and less than minivans.

(D) The prediction will be refined after an analysis of the sales in the first quarter of this year.

(E) The number of models of compacts cars that Tecumseh produces is greater than the number of models of either minivans or SUVs.

 

2) Most people can gain vitamin C from fruits such as oranges and cantaloupes.  People with Laestrygonian Disease have weakened digestive systems that cannot digest fruit or vitamin supplements.  The easiest foods for these people to digest are grains such as rice and barley.  Regular intake of vitamin C would be extremely beneficial to those who suffer from Laestrygonian Disease, so scientists have figured out a way to create “fortified rice” by infusing rice with high doses of vitamin C.  This fortified rice will provide great benefit to those with Laestrygonian Disease.

Which one of the following is an assumption on which the conclusion depends?

(A) Eventually, this fortified rice will be the optimal way for most people to have a regular intake of vitamin C.

(B) The problems that folks with Laestrygonian Disease have digesting fruit are different from their problems digesting vitamin supplements.

(C) People with Laestrygonian Disease will not be unable to assimilate the form of vitamin C that is present in the fortified rice.

(D) Only people whose diets consist largely of grains would be able to derive benefit from the vitamin C in the fortified rice.

(E) Vitamin C is the only nutrient which can be infused into rice in such high quantities without compromising the nutritional integrity of the vitamin.

 

3) Sale Analyst: When polled, all consumers consistently say that, for household staples, they would buy the lowest cost items.  Even when other factors, such as inherent product quality, are introduce, all consumers still argue that low cost should be the highest priority in buying household staples.  Therefore, these responses demonstrate how little most people are aware of the actual priorities that drive their purchasing decisions.

The conclusion above is properly drawn if which of the following is assumed?

(A) Many people on restricted incomes are not able to afford any brands of household staples more expensive than the lowest cost items.

(B) Consumers do not always have the motivation to be truthful about their real motivations when responding to a stranger giving a poll.

(C) Often, higher priced brands of household staples, especially organic versions, are more nutritious and are rated better-tasting in double-blind tests.

(D) People often give unreliable and self-contradictory explanations after the impulse purchase of a snack or dessert.

(E) For many household staples, famous name brands with recognizable ads consistently outsell less expensive brands of the same products.

 

4) Urban planner: The mayor of Dismaston supports a new tax code that would assess local businesses on the property value of their site rather than on their income or profits; the mayor argues that this change will not contribute to any loss in tax revenue for the city.  Several city council members disagree, citing similar changes to tax code that were unsuccessful in cities similar to Dismaston.  The council members’ argument is without merit, though, because property values rise steadily each year, while business incomes fluctuate wildly with the national economy.

Which of the following is an assumption on which the argument depends?

(A) The new tax code would not apply to the significant number of online business associated with the city that have no physical presence on a piece of property in the city.

(B) In previous years, the successes and failures of many previous Dismaston tax codes to generate income have been mirrored by similar tax codes in the the same cities similar to Dismaston in other respects.

(C) A small number of store fronts in the downtown neighborhood are vacant: under the new code, these businesses are likely to owe higher taxes than they have paid, unless some loophole is written into the law.

(D) The annual percentage rise in real estate values in Dismaston has been consistently more than the average annual growth rate percentages across all businesses with properties in Dismaston.

(E) In any year, some unsuccessful businesses will close and other businesses, some quite promising, will open, but total amount of property in the city is fixed and unchanging, providing greater stability.

Detailed explanations will come at the end of this article.

 

Assumptions
Assumptions are the linchpins of arguments, the glue that binds together the evidence and the conclusion.   Ideally, assumptions are what make the conclusion follow logically from the evidence.  I have already written about assumptions in CR arguments in two blogs:

1) Arguments and Assumptions on the GMAT

2) Assumptions and the Negation Test on the GMAT

The skill of isolating an assumption is also important because affirming the truth of an assumption is one way to strengthen an argument and attacking the assumption is one way to weaken an argument.

If you are not familiar with the Negation Test, then I would definitely recommend studying this powerful technique for isolating assumptions of arguments.

 

Summary
If you had any insights reading this blog or the linked blogs, you may want to revisit the questions at the top before jumping into the explanations below.  If you have discovered your own strategies for analyzing the assumptions of arguments, please feel free to let us know in the comments section!



 

Practice problem explanations
1) The credited answer is (C).  Tecumseh predicts the best-selling vehicle categories this year will be, in order, (1) compact cars, (2) minivans, and (3) SUVs.  Since they used last year’s sale to make these prediction, those three categories must have been first, second, and third in last year’s sale.  Sales are probably based on total revenue, but because every model of car generates approximately the same amount of profit, the order in sales would be the same as the order in profit.  Thus, last year, compact cars generated the most profit, minivans the second most, and SUVs the third most.  Thus, no other car had profits between minivans and SUVs.

The scope of (A) is too wide.  The most popular cars on the road would include many makes, not just cars from Tecumseh.  What is true for other manufacturers may or may not be true for Tecumseh. This choice is incorrect.

Choice (B) is not necessarily true.  It could be true that some models were radically enhanced or changed.  We don’t know how this year’s sales actually will proceed.  All we know is Tecumseh’s prediction, and for Tecumseh to have made this particular prediction, we need not know anything about continuity or change in any model.  This choice is incorrect.

Choice (D) might be a good idea, but this would be well after the fact, well after the prediction.  In this argument, we are not as interested in whether the prediction will be true: we are simply interested in how the marketing analysts arrived at this particular prediction.  What they do afterwards might be great, but it doesn’t play into the logic of how they made the prediction.  This choice is incorrect.

Choice (E) is not precise.  The total number of compact cars that Tecumseh sold last year must be greater than the total number of minivans, but we have no information about how many models Tecumseh has in each category.  Suppose they had one one model of compact car, and sold a thousand of them, and suppose they had twelve models of minivans and sold ten of each.  They would have sold many more compact cars, and have generated much more in profits in that category, despite the fact that they have only one model.  This choice is incorrect.

 

2) The credited answer is (C).  If folks with Laestrygonian Disease cannot assimilate the Vitamin C in the rice, then it won’t help them, and eating the fortified rice will not provide them any particular benefit.  If we negate this option, it shatters the argument.  This is a true assumption.

(A) This may be true, although I am skeptical that any human-made improved food would be better than the fruits designed by Nature!  Regardless, whether this is true or not does not have any bearing on how helpful the fortified rice will be for the folks with Laestrygonian Disease.  This option is incorrect.

(B) This is intriguing.  Let’s negate this.  Suppose it were the exact same problem, say, the exact same missing enzyme, that made it impossible to digest both fruit and vitamin supplements.  Then what?  Would that mean they also couldn’t digest the fortified rice, or get the vitamin C they need from it?  We cannot say.  It’s conceivable that the argument could still work, so negating this does not destroy the argument.  This is not an assumption.

(D) Let’s negate this.  Suppose the fortified rice benefits everyone—even the no-carbs fanatic who hasn’t touched carbs in a decade: even when this person breaks his carb-fast and has the fortified rice, he has benefit from it.  What then?  Whether these other people benefit or not from the fortified rice has no bearing on whether it helps the folks with Laestrygonian Disease.  This choice is incorrect.

(E) Let’s negate this.  Suppose we can infused dozens of other vitamins and minerals into the rice, all with high nutritional yield.  That would only be good for the folks with Laestrygonian Disease—the more vitamins, the better!  It certainly would not impact whether these folks derived any benefit from the vitamin C in the rice.  This choice is incorrect.

 

3) This argument contains an interesting leap.  Folks say they will buy the cheapest items, therefore, they aren’t aware of their actual purchasing patterns.  This means, the sales analyst must be assuming that many consumers buy brands of household staples that are not the cheapest.

The credited answer is (E).  If famous brand of household staples are more expensive yet outselling the cheaper no-name brand, then it must mean that the vast majority of consumers are buying them.  This behavior belies their stated priorities.  This would justify the Sale Analyst’s conclusion about the divergence of their stated priorities and their actual purchases.

(A) This is a relatively small segment of the population, and it would not explain the macro-patterns that the author is addressing.  Also, keep in mind, the difference in price between the cheapest brand of a household staple and a more expensive name-brand might be under 50¢.  Even for someone on Food Stamps, if that person really wanted the more expensive brand on that signal item, they probably could afford it.  For a few reasons, this answer does not work.

(B) Let’s negate this.  Suppose consumers are 100% swear-on-the-Bible truthful in what they tell pollsters.  If anything, this would strengthen the Sales Analyst’s argument: if consumers entirely candid in saying they will buy one thing, and then they go and buy something else, they really aren’t aware of their shopping priorities!  Negating this could strengthen the argument, so this is not an assumption.

(C) This is tempting.  Certainly if these more expensive brands are organic and nutritious and yummy, there would be a number of reasons that people might buy them.  BUT, do we know that people will actually shell out the extra money to buy them?  Maybe, maybe not.  If we had evidence that consumers actually were purchasing these more expensive brands, that would be a compelling assumption: unfortunately, as is, this is only suggestive.  This choice is incorrect.

(D) This is suggestive.  Suppose explanations after an impulse purchase are unreliable.  Does this mean that explanations before the purchase of a household staple would also be unreliable?  Maybe, maybe not.  This might suggest some kind of pattern, but we don’t have the evidence to conclude anything.  This option is incorrect.

 

4) This is a very tricky argument.  The mayor has a new tax code: tax business on property value, not on business income.  (In real world terms, these seems a most dubious plan!)  Some city council members disagree for various reasons.  The actual argument that concerns us is the speaker’s argument, the argument of the Urban Planner.  The Urban planner disagrees with these city council members, so in essence, he agrees with the mayor and thinks the mayor’s plan is a good one.  The urban planner’s stated reason is that business income fluctuates wildly, and therefore it must not be as dependable a source of income via taxes as are the steadily rising levels of property values.  In other words, taxes on property will go steadily up, and therefore will make more money than the up-and-down yields from business income taxes.

The credited answer is (D): Let’s negate this.  Suppose the annual percentage rise in real estate values, in property values, has been consistent less than the average annual growth rate percentages across all businesses with properties in Dismaston.  Thus, even though individual business may be fluctuating, when we average over all businesses, the growth rate is higher for that average than is for property values.  Thus, revenue from taxes on business income would grow faster than revenue from taxes on property.  If this negation is true, it devastates the mayor’s position, and therefore devastates the position that the Urban Planner is taking.  Because the negation is so destructive, the un-negated version that appears in choice (D) must be an assumption.

(A) Presumably, business that have no physical presence would not fall under this new tax code; if all these business with no physical property went from their current tax to zero, that would be a major income loss, and such a loss presumably would be obvious to the mayor who claims that there will be no loss.  What tax code applies to these online businesses is unclear, but this is out of scope.  This choice is incorrect.

(B) This is suggestive of the conclusion of the council member’s argument.   If other Dismaston tax codes have fared about as well as similar codes in similar cities, we might expect the same for the current code.  This is suggestive, but not conclusive, and it supports the wrong argument, the council member’s argument, not the urban planner’s argument.   This choice is incorrect.

(C) If there’s no loophole, this would support the mayor’s plan, which would support the urban planners argument.  This is a support, but not an assumption.  Even if the loophole is written and the small number of vacant storefront pay nothing, this does not necessarily mean anything about the success or failure of the mayor’s plan.  This choice is incorrect.

(E) Well, if we totally negate this, then it would mean that there’s total business stability in Dismaston: the same businesses now have all been in business for 50 years, and all will continue in business for at least another fifty years.  It’s not particularly clear if this makes any difference at all to the mayor’s plan.  Since negating it doesn’t destroy the mayor’s plan, this is not an assumption.  This choice is incorrect.

 

The post GMAT Critical Reasoning: More on Assumption Questions appeared first on Magoosh GMAT Blog.
This Blog post was imported into the forum automatically. We hope you found it helpful. Please use the Kudos button if you did, or please PM/DM me if you found it disruptive and I will take care of it. -BB
Magoosh GMAT Instructor
Joined: 28 Dec 2011
Posts: 4490
Own Kudos [?]: 28872 [0]
Given Kudos: 130
GMAT Critical Reasoning: More on Assumption Questions [#permalink]
Expert Reply
FROM Magoosh Blog: GMAT Critical Reasoning: More on Assumption Questions
First, four practice CR questions asking for the assumptions of arguments.

1) In anticipation of the coming year, Tecumseh Autos, a national auto manufacturer, is anticipating sales of its vehicles.  Tecumseh manufactures compact cars, sedans, minivans, trucks, SUVs, and sports cars.  In all categories of vehicles, Tecumseh sets prices so that the profit per vehicle is, on average, about the same.   Since the best indicator for sales in each category are the sales last year, Tecumseh’s marketing analysts’ prediction of the three most profitable categories of vehicles in the coming year will be compact cars, minivans, and SUVs respectively.

Tecumseh’s marketing analysts’ prediction relies on which one of the following assumptions?

(A) Across all manufacturers, the most popular cars on the road in America are compact cars, minivans, and SUVs.

(B) The models of Tecumseh’s compact cars to be sold in the upcoming year are identical to or similar to those of last year.

(C) Last year, no other category of Tecumseh’s vehicles generated more profits than SUVs and less than minivans.

(D) The prediction will be refined after an analysis of the sales in the first quarter of this year.

(E) The number of models of compacts cars that Tecumseh produces is greater than the number of models of either minivans or SUVs.

 

2) Most people can gain vitamin C from fruits such as oranges and cantaloupes.  People with Laestrygonian Disease have weakened digestive systems that cannot digest fruit or vitamin supplements.  The easiest foods for these people to digest are grains such as rice and barley.  Regular intake of vitamin C would be extremely beneficial to those who suffer from Laestrygonian Disease, so scientists have figured out a way to create “fortified rice” by infusing rice with high doses of vitamin C.  This fortified rice will provide great benefit to those with Laestrygonian Disease.

Which one of the following is an assumption on which the conclusion depends?

(A) Eventually, this fortified rice will be the optimal way for most people to have a regular intake of vitamin C.

(B) The problems that folks with Laestrygonian Disease have digesting fruit are different from their problems digesting vitamin supplements.

(C) People with Laestrygonian Disease will not be unable to assimilate the form of vitamin C that is present in the fortified rice.

(D) Only people whose diets consist largely of grains would be able to derive benefit from the vitamin C in the fortified rice.

(E) Vitamin C is the only nutrient which can be infused into rice in such high quantities without compromising the nutritional integrity of the vitamin.

 

3) Sale Analyst: When polled, all consumers consistently say that, for household staples, they would buy the lowest cost items.  Even when other factors, such as inherent product quality, are introduce, all consumers still argue that low cost should be the highest priority in buying household staples.  Therefore, these responses demonstrate how little most people are aware of the actual priorities that drive their purchasing decisions.

The conclusion above is properly drawn if which of the following is assumed?

(A) Many people on restricted incomes are not able to afford any brands of household staples more expensive than the lowest cost items.

(B) Consumers do not always have the motivation to be truthful about their real motivations when responding to a stranger giving a poll.

(C) Often, higher priced brands of household staples, especially organic versions, are more nutritious and are rated better-tasting in double-blind tests.

(D) People often give unreliable and self-contradictory explanations after the impulse purchase of a snack or dessert.

(E) For many household staples, famous name brands with recognizable ads consistently outsell less expensive brands of the same products.

 

4) Urban planner: The mayor of Dismaston supports a new tax code that would assess local businesses on the property value of their site rather than on their income or profits; the mayor argues that this change will not contribute to any loss in tax revenue for the city.  Several city council members disagree, citing similar changes to tax code that were unsuccessful in cities similar to Dismaston.  The council members’ argument is without merit, though, because property values rise steadily each year, while business incomes fluctuate wildly with the national economy.

Which of the following is an assumption on which the argument depends?

(A) The new tax code would not apply to the significant number of online business associated with the city that have no physical presence on a piece of property in the city.

(B) In previous years, the successes and failures of many previous Dismaston tax codes to generate income have been mirrored by similar tax codes in the the same cities similar to Dismaston in other respects.

(C) A small number of store fronts in the downtown neighborhood are vacant: under the new code, these businesses are likely to owe higher taxes than they have paid, unless some loophole is written into the law.

(D) The annual percentage rise in real estate values in Dismaston has been consistently more than the average annual growth rate percentages across all businesses with properties in Dismaston.

(E) In any year, some unsuccessful businesses will close and other businesses, some quite promising, will open, but total amount of property in the city is fixed and unchanging, providing greater stability.

Detailed explanations will come at the end of this article.

 

Assumptions
Assumptions are the linchpins of arguments, the glue that binds together the evidence and the conclusion.   Ideally, assumptions are what make the conclusion follow logically from the evidence.  I have already written about assumptions in CR arguments in two blogs:

1) Arguments and Assumptions on the GMAT

2) Assumptions and the Negation Test on the GMAT

The skill of isolating an assumption is also important because affirming the truth of an assumption is one way to strengthen an argument and attacking the assumption is one way to weaken an argument.

If you are not familiar with the Negation Test, then I would definitely recommend studying this powerful technique for isolating assumptions of arguments.

 

Summary
If you had any insights reading this blog or the linked blogs, you may want to revisit the questions at the top before jumping into the explanations below.  If you have discovered your own strategies for analyzing the assumptions of arguments, please feel free to let us know in the comments section!



 

Practice problem explanations
1) The credited answer is (C).  Tecumseh predicts the best-selling vehicle categories this year will be, in order, (1) compact cars, (2) minivans, and (3) SUVs.  Since they used last year’s sale to make these prediction, those three categories must have been first, second, and third in last year’s sale.  Sales are probably based on total revenue, but because every model of car generates approximately the same amount of profit, the order in sales would be the same as the order in profit.  Thus, last year, compact cars generated the most profit, minivans the second most, and SUVs the third most.  Thus, no other car had profits between minivans and SUVs.

The scope of (A) is too wide.  The most popular cars on the road would include many makes, not just cars from Tecumseh.  What is true for other manufacturers may or may not be true for Tecumseh. This choice is incorrect.

Choice (B) is not necessarily true.  It could be true that some models were radically enhanced or changed.  We don’t know how this year’s sales actually will proceed.  All we know is Tecumseh’s prediction, and for Tecumseh to have made this particular prediction, we need not know anything about continuity or change in any model.  This choice is incorrect.

Choice (D) might be a good idea, but this would be well after the fact, well after the prediction.  In this argument, we are not as interested in whether the prediction will be true: we are simply interested in how the marketing analysts arrived at this particular prediction.  What they do afterwards might be great, but it doesn’t play into the logic of how they made the prediction.  This choice is incorrect.

Choice (E) is not precise.  The total number of compact cars that Tecumseh sold last year must be greater than the total number of minivans, but we have no information about how many models Tecumseh has in each category.  Suppose they had one one model of compact car, and sold a thousand of them, and suppose they had twelve models of minivans and sold ten of each.  They would have sold many more compact cars, and have generated much more in profits in that category, despite the fact that they have only one model.  This choice is incorrect.

 

2) The credited answer is (C).  If folks with Laestrygonian Disease cannot assimilate the Vitamin C in the rice, then it won’t help them, and eating the fortified rice will not provide them any particular benefit.  If we negate this option, it shatters the argument.  This is a true assumption.

(A) This may be true, although I am skeptical that any human-made improved food would be better than the fruits designed by Nature!  Regardless, whether this is true or not does not have any bearing on how helpful the fortified rice will be for the folks with Laestrygonian Disease.  This option is incorrect.

(B) This is intriguing.  Let’s negate this.  Suppose it were the exact same problem, say, the exact same missing enzyme, that made it impossible to digest both fruit and vitamin supplements.  Then what?  Would that mean they also couldn’t digest the fortified rice, or get the vitamin C they need from it?  We cannot say.  It’s conceivable that the argument could still work, so negating this does not destroy the argument.  This is not an assumption.

(D) Let’s negate this.  Suppose the fortified rice benefits everyone—even the no-carbs fanatic who hasn’t touched carbs in a decade: even when this person breaks his carb-fast and has the fortified rice, he has benefit from it.  What then?  Whether these other people benefit or not from the fortified rice has no bearing on whether it helps the folks with Laestrygonian Disease.  This choice is incorrect.

(E) Let’s negate this.  Suppose we can infused dozens of other vitamins and minerals into the rice, all with high nutritional yield.  That would only be good for the folks with Laestrygonian Disease—the more vitamins, the better!  It certainly would not impact whether these folks derived any benefit from the vitamin C in the rice.  This choice is incorrect.

 

3) This argument contains an interesting leap.  Folks say they will buy the cheapest items, therefore, they aren’t aware of their actual purchasing patterns.  This means, the sales analyst must be assuming that many consumers buy brands of household staples that are not the cheapest.

The credited answer is (E).  If famous brand of household staples are more expensive yet outselling the cheaper no-name brand, then it must mean that the vast majority of consumers are buying them.  This behavior belies their stated priorities.  This would justify the Sale Analyst’s conclusion about the divergence of their stated priorities and their actual purchases.

(A) This is a relatively small segment of the population, and it would not explain the macro-patterns that the author is addressing.  Also, keep in mind, the difference in price between the cheapest brand of a household staple and a more expensive name-brand might be under 50¢.  Even for someone on Food Stamps, if that person really wanted the more expensive brand on that signal item, they probably could afford it.  For a few reasons, this answer does not work.

(B) Let’s negate this.  Suppose consumers are 100% swear-on-the-Bible truthful in what they tell pollsters.  If anything, this would strengthen the Sales Analyst’s argument: if consumers entirely candid in saying they will buy one thing, and then they go and buy something else, they really aren’t aware of their shopping priorities!  Negating this could strengthen the argument, so this is not an assumption.

(C) This is tempting.  Certainly if these more expensive brands are organic and nutritious and yummy, there would be a number of reasons that people might buy them.  BUT, do we know that people will actually shell out the extra money to buy them?  Maybe, maybe not.  If we had evidence that consumers actually were purchasing these more expensive brands, that would be a compelling assumption: unfortunately, as is, this is only suggestive.  This choice is incorrect.

(D) This is suggestive.  Suppose explanations after an impulse purchase are unreliable.  Does this mean that explanations before the purchase of a household staple would also be unreliable?  Maybe, maybe not.  This might suggest some kind of pattern, but we don’t have the evidence to conclude anything.  This option is incorrect.

 

4) This is a very tricky argument.  The mayor has a new tax code: tax business on property value, not on business income.  (In real world terms, these seems a most dubious plan!)  Some city council members disagree for various reasons.  The actual argument that concerns us is the speaker’s argument, the argument of the Urban Planner.  The Urban planner disagrees with these city council members, so in essence, he agrees with the mayor and thinks the mayor’s plan is a good one.  The urban planner’s stated reason is that business income fluctuates wildly, and therefore it must not be as dependable a source of income via taxes as are the steadily rising levels of property values.  In other words, taxes on property will go steadily up, and therefore will make more money than the up-and-down yields from business income taxes.

The credited answer is (D): Let’s negate this.  Suppose the annual percentage rise in real estate values, in property values, has been consistent less than the average annual growth rate percentages across all businesses with properties in Dismaston.  Thus, even though individual business may be fluctuating, when we average over all businesses, the growth rate is higher for that average than is for property values.  Thus, revenue from taxes on business income would grow faster than revenue from taxes on property.  If this negation is true, it devastates the mayor’s position, and therefore devastates the position that the Urban Planner is taking.  Because the negation is so destructive, the un-negated version that appears in choice (D) must be an assumption.

(A) Presumably, business that have no physical presence would not fall under this new tax code; if all these business with no physical property went from their current tax to zero, that would be a major income loss, and such a loss presumably would be obvious to the mayor who claims that there will be no loss.  What tax code applies to these online businesses is unclear, but this is out of scope.  This choice is incorrect.

(B) This is suggestive of the conclusion of the council member’s argument.   If other Dismaston tax codes have fared about as well as similar codes in similar cities, we might expect the same for the current code.  This is suggestive, but not conclusive, and it supports the wrong argument, the council member’s argument, not the urban planner’s argument.   This choice is incorrect.

(C) If there’s no loophole, this would support the mayor’s plan, which would support the urban planners argument.  This is a support, but not an assumption.  Even if the loophole is written and the small number of vacant storefront pay nothing, this does not necessarily mean anything about the success or failure of the mayor’s plan.  This choice is incorrect.

(E) Well, if we totally negate this, then it would mean that there’s total business stability in Dismaston: the same businesses now have all been in business for 50 years, and all will continue in business for at least another fifty years.  It’s not particularly clear if this makes any difference at all to the mayor’s plan.  Since negating it doesn’t destroy the mayor’s plan, this is not an assumption.  This choice is incorrect.

 

The post GMAT Critical Reasoning: More on Assumption Questions appeared first on Magoosh GMAT Blog.
This Blog post was imported into the forum automatically. We hope you found it helpful. Please use the Kudos button if you did, or please PM/DM me if you found it disruptive and I will take care of it. -BB
Magoosh GMAT Instructor
Joined: 28 Dec 2011
Posts: 4490
Own Kudos [?]: 28872 [0]
Given Kudos: 130
GMAT Critical Reasoning: More on Assumption Questions [#permalink]
Expert Reply
FROM Magoosh Blog: GMAT Critical Reasoning: More on Assumption Questions
First, four practice CR questions asking for the assumptions of arguments.

1) In anticipation of the coming year, Tecumseh Autos, a national auto manufacturer, is anticipating sales of its vehicles.  Tecumseh manufactures compact cars, sedans, minivans, trucks, SUVs, and sports cars.  In all categories of vehicles, Tecumseh sets prices so that the profit per vehicle is, on average, about the same.   Since the best indicator for sales in each category are the sales last year, Tecumseh’s marketing analysts’ prediction of the three most profitable categories of vehicles in the coming year will be compact cars, minivans, and SUVs respectively.

Tecumseh’s marketing analysts’ prediction relies on which one of the following assumptions?

(A) Across all manufacturers, the most popular cars on the road in America are compact cars, minivans, and SUVs.

(B) The models of Tecumseh’s compact cars to be sold in the upcoming year are identical to or similar to those of last year.

(C) Last year, no other category of Tecumseh’s vehicles generated more profits than SUVs and less than minivans.

(D) The prediction will be refined after an analysis of the sales in the first quarter of this year.

(E) The number of models of compacts cars that Tecumseh produces is greater than the number of models of either minivans or SUVs.

 

2) Most people can gain vitamin C from fruits such as oranges and cantaloupes.  People with Laestrygonian Disease have weakened digestive systems that cannot digest fruit or vitamin supplements.  The easiest foods for these people to digest are grains such as rice and barley.  Regular intake of vitamin C would be extremely beneficial to those who suffer from Laestrygonian Disease, so scientists have figured out a way to create “fortified rice” by infusing rice with high doses of vitamin C.  This fortified rice will provide great benefit to those with Laestrygonian Disease.

Which one of the following is an assumption on which the conclusion depends?

(A) Eventually, this fortified rice will be the optimal way for most people to have a regular intake of vitamin C.

(B) The problems that folks with Laestrygonian Disease have digesting fruit are different from their problems digesting vitamin supplements.

(C) People with Laestrygonian Disease will not be unable to assimilate the form of vitamin C that is present in the fortified rice.

(D) Only people whose diets consist largely of grains would be able to derive benefit from the vitamin C in the fortified rice.

(E) Vitamin C is the only nutrient which can be infused into rice in such high quantities without compromising the nutritional integrity of the vitamin.

 

3) Sale Analyst: When polled, all consumers consistently say that, for household staples, they would buy the lowest cost items.  Even when other factors, such as inherent product quality, are introduce, all consumers still argue that low cost should be the highest priority in buying household staples.  Therefore, these responses demonstrate how little most people are aware of the actual priorities that drive their purchasing decisions.

The conclusion above is properly drawn if which of the following is assumed?

(A) Many people on restricted incomes are not able to afford any brands of household staples more expensive than the lowest cost items.

(B) Consumers do not always have the motivation to be truthful about their real motivations when responding to a stranger giving a poll.

(C) Often, higher priced brands of household staples, especially organic versions, are more nutritious and are rated better-tasting in double-blind tests.

(D) People often give unreliable and self-contradictory explanations after the impulse purchase of a snack or dessert.

(E) For many household staples, famous name brands with recognizable ads consistently outsell less expensive brands of the same products.

 

4) Urban planner: The mayor of Dismaston supports a new tax code that would assess local businesses on the property value of their site rather than on their income or profits; the mayor argues that this change will not contribute to any loss in tax revenue for the city.  Several city council members disagree, citing similar changes to tax code that were unsuccessful in cities similar to Dismaston.  The council members’ argument is without merit, though, because property values rise steadily each year, while business incomes fluctuate wildly with the national economy.

Which of the following is an assumption on which the argument depends?

(A) The new tax code would not apply to the significant number of online business associated with the city that have no physical presence on a piece of property in the city.

(B) In previous years, the successes and failures of many previous Dismaston tax codes to generate income have been mirrored by similar tax codes in the the same cities similar to Dismaston in other respects.

(C) A small number of store fronts in the downtown neighborhood are vacant: under the new code, these businesses are likely to owe higher taxes than they have paid, unless some loophole is written into the law.

(D) The annual percentage rise in real estate values in Dismaston has been consistently more than the average annual growth rate percentages across all businesses with properties in Dismaston.

(E) In any year, some unsuccessful businesses will close and other businesses, some quite promising, will open, but total amount of property in the city is fixed and unchanging, providing greater stability.

Detailed explanations will come at the end of this article.

 

Assumptions
Assumptions are the linchpins of arguments, the glue that binds together the evidence and the conclusion.   Ideally, assumptions are what make the conclusion follow logically from the evidence.  I have already written about assumptions in CR arguments in two blogs:

1) Arguments and Assumptions on the GMAT

2) Assumptions and the Negation Test on the GMAT

The skill of isolating an assumption is also important because affirming the truth of an assumption is one way to strengthen an argument and attacking the assumption is one way to weaken an argument.

If you are not familiar with the Negation Test, then I would definitely recommend studying this powerful technique for isolating assumptions of arguments.

 

Summary
If you had any insights reading this blog or the linked blogs, you may want to revisit the questions at the top before jumping into the explanations below.  If you have discovered your own strategies for analyzing the assumptions of arguments, please feel free to let us know in the comments section!



 

Practice problem explanations
1) The credited answer is (C).  Tecumseh predicts the best-selling vehicle categories this year will be, in order, (1) compact cars, (2) minivans, and (3) SUVs.  Since they used last year’s sale to make these prediction, those three categories must have been first, second, and third in last year’s sale.  Sales are probably based on total revenue, but because every model of car generates approximately the same amount of profit, the order in sales would be the same as the order in profit.  Thus, last year, compact cars generated the most profit, minivans the second most, and SUVs the third most.  Thus, no other car had profits between minivans and SUVs.

The scope of (A) is too wide.  The most popular cars on the road would include many makes, not just cars from Tecumseh.  What is true for other manufacturers may or may not be true for Tecumseh. This choice is incorrect.

Choice (B) is not necessarily true.  It could be true that some models were radically enhanced or changed.  We don’t know how this year’s sales actually will proceed.  All we know is Tecumseh’s prediction, and for Tecumseh to have made this particular prediction, we need not know anything about continuity or change in any model.  This choice is incorrect.

Choice (D) might be a good idea, but this would be well after the fact, well after the prediction.  In this argument, we are not as interested in whether the prediction will be true: we are simply interested in how the marketing analysts arrived at this particular prediction.  What they do afterwards might be great, but it doesn’t play into the logic of how they made the prediction.  This choice is incorrect.

Choice (E) is not precise.  The total number of compact cars that Tecumseh sold last year must be greater than the total number of minivans, but we have no information about how many models Tecumseh has in each category.  Suppose they had one one model of compact car, and sold a thousand of them, and suppose they had twelve models of minivans and sold ten of each.  They would have sold many more compact cars, and have generated much more in profits in that category, despite the fact that they have only one model.  This choice is incorrect.

 

2) The credited answer is (C).  If folks with Laestrygonian Disease cannot assimilate the Vitamin C in the rice, then it won’t help them, and eating the fortified rice will not provide them any particular benefit.  If we negate this option, it shatters the argument.  This is a true assumption.

(A) This may be true, although I am skeptical that any human-made improved food would be better than the fruits designed by Nature!  Regardless, whether this is true or not does not have any bearing on how helpful the fortified rice will be for the folks with Laestrygonian Disease.  This option is incorrect.

(B) This is intriguing.  Let’s negate this.  Suppose it were the exact same problem, say, the exact same missing enzyme, that made it impossible to digest both fruit and vitamin supplements.  Then what?  Would that mean they also couldn’t digest the fortified rice, or get the vitamin C they need from it?  We cannot say.  It’s conceivable that the argument could still work, so negating this does not destroy the argument.  This is not an assumption.

(D) Let’s negate this.  Suppose the fortified rice benefits everyone—even the no-carbs fanatic who hasn’t touched carbs in a decade: even when this person breaks his carb-fast and has the fortified rice, he has benefit from it.  What then?  Whether these other people benefit or not from the fortified rice has no bearing on whether it helps the folks with Laestrygonian Disease.  This choice is incorrect.

(E) Let’s negate this.  Suppose we can infused dozens of other vitamins and minerals into the rice, all with high nutritional yield.  That would only be good for the folks with Laestrygonian Disease—the more vitamins, the better!  It certainly would not impact whether these folks derived any benefit from the vitamin C in the rice.  This choice is incorrect.

 

3) This argument contains an interesting leap.  Folks say they will buy the cheapest items, therefore, they aren’t aware of their actual purchasing patterns.  This means, the sales analyst must be assuming that many consumers buy brands of household staples that are not the cheapest.

The credited answer is (E).  If famous brand of household staples are more expensive yet outselling the cheaper no-name brand, then it must mean that the vast majority of consumers are buying them.  This behavior belies their stated priorities.  This would justify the Sale Analyst’s conclusion about the divergence of their stated priorities and their actual purchases.

(A) This is a relatively small segment of the population, and it would not explain the macro-patterns that the author is addressing.  Also, keep in mind, the difference in price between the cheapest brand of a household staple and a more expensive name-brand might be under 50¢.  Even for someone on Food Stamps, if that person really wanted the more expensive brand on that signal item, they probably could afford it.  For a few reasons, this answer does not work.

(B) Let’s negate this.  Suppose consumers are 100% swear-on-the-Bible truthful in what they tell pollsters.  If anything, this would strengthen the Sales Analyst’s argument: if consumers entirely candid in saying they will buy one thing, and then they go and buy something else, they really aren’t aware of their shopping priorities!  Negating this could strengthen the argument, so this is not an assumption.

(C) This is tempting.  Certainly if these more expensive brands are organic and nutritious and yummy, there would be a number of reasons that people might buy them.  BUT, do we know that people will actually shell out the extra money to buy them?  Maybe, maybe not.  If we had evidence that consumers actually were purchasing these more expensive brands, that would be a compelling assumption: unfortunately, as is, this is only suggestive.  This choice is incorrect.

(D) This is suggestive.  Suppose explanations after an impulse purchase are unreliable.  Does this mean that explanations before the purchase of a household staple would also be unreliable?  Maybe, maybe not.  This might suggest some kind of pattern, but we don’t have the evidence to conclude anything.  This option is incorrect.

 

4) This is a very tricky argument.  The mayor has a new tax code: tax business on property value, not on business income.  (In real world terms, these seems a most dubious plan!)  Some city council members disagree for various reasons.  The actual argument that concerns us is the speaker’s argument, the argument of the Urban Planner.  The Urban planner disagrees with these city council members, so in essence, he agrees with the mayor and thinks the mayor’s plan is a good one.  The urban planner’s stated reason is that business income fluctuates wildly, and therefore it must not be as dependable a source of income via taxes as are the steadily rising levels of property values.  In other words, taxes on property will go steadily up, and therefore will make more money than the up-and-down yields from business income taxes.

The credited answer is (D): Let’s negate this.  Suppose the annual percentage rise in real estate values, in property values, has been consistent less than the average annual growth rate percentages across all businesses with properties in Dismaston.  Thus, even though individual business may be fluctuating, when we average over all businesses, the growth rate is higher for that average than is for property values.  Thus, revenue from taxes on business income would grow faster than revenue from taxes on property.  If this negation is true, it devastates the mayor’s position, and therefore devastates the position that the Urban Planner is taking.  Because the negation is so destructive, the un-negated version that appears in choice (D) must be an assumption.

(A) Presumably, business that have no physical presence would not fall under this new tax code; if all these business with no physical property went from their current tax to zero, that would be a major income loss, and such a loss presumably would be obvious to the mayor who claims that there will be no loss.  What tax code applies to these online businesses is unclear, but this is out of scope.  This choice is incorrect.

(B) This is suggestive of the conclusion of the council member’s argument.   If other Dismaston tax codes have fared about as well as similar codes in similar cities, we might expect the same for the current code.  This is suggestive, but not conclusive, and it supports the wrong argument, the council member’s argument, not the urban planner’s argument.   This choice is incorrect.

(C) If there’s no loophole, this would support the mayor’s plan, which would support the urban planners argument.  This is a support, but not an assumption.  Even if the loophole is written and the small number of vacant storefront pay nothing, this does not necessarily mean anything about the success or failure of the mayor’s plan.  This choice is incorrect.

(E) Well, if we totally negate this, then it would mean that there’s total business stability in Dismaston: the same businesses now have all been in business for 50 years, and all will continue in business for at least another fifty years.  It’s not particularly clear if this makes any difference at all to the mayor’s plan.  Since negating it doesn’t destroy the mayor’s plan, this is not an assumption.  This choice is incorrect.

 

The post GMAT Critical Reasoning: More on Assumption Questions appeared first on Magoosh GMAT Blog.
This Blog post was imported into the forum automatically. We hope you found it helpful. Please use the Kudos button if you did, or please PM/DM me if you found it disruptive and I will take care of it. -BB
Magoosh GMAT Instructor
Joined: 28 Dec 2011
Posts: 4490
Own Kudos [?]: 28872 [0]
Given Kudos: 130
GMAT Tuesday: Common AWA Flaws – Vague Words [#permalink]
Expert Reply
FROM Magoosh Blog: GMAT Tuesday: Common AWA Flaws – Vague Words
The Analytical Writing Assessment (AWA) for the GMAT can catch you off guard if you aren’t ready. If you aren’t familiar with the structure of arguments or you are familiar with common argument flaws, you may find yourself stressed and wasting time on test day.



In this video, we’ll look at vague words and how they lead to weak conclusions. I’ll talk about how to identify these words and what to do when you see them.

We look at an example prompt, which you can find in the PDF of sample prompts on the mba.com website. Here’s a link to the page where you can download the example prompts.

Check out this week’s board:


The post GMAT Tuesday: Common AWA Flaws – Vague Words appeared first on Magoosh GMAT Blog.
This Blog post was imported into the forum automatically. We hope you found it helpful. Please use the Kudos button if you did, or please PM/DM me if you found it disruptive and I will take care of it. -BB
Magoosh GMAT Instructor
Joined: 28 Dec 2011
Posts: 4490
Own Kudos [?]: 28872 [0]
Given Kudos: 130
The 3 Types of B-Schools You Should Apply To [#permalink]
Expert Reply
FROM Magoosh Blog: The 3 Types of B-Schools You Should Apply To
It’s never smart to put all your eggs in one basket – aim too high and you may be left with nowhere to go, aim too low and you might get acceptances to schools that aren’t quite right for you. For the highest chances of getting in AND for the best MBA experience, you need to evaluate your competitiveness and apply to the best MBA programs for you, spread out over the following three categories: reasonable reaches, on-pars, and safety schools.

Defining Our Terms
Reasonable reaches: These are the schools for which your acceptance is unlikely but possible, especially with an impressive application.
On-pars: You are competitive at these schools and with a great application, you have a solid chance of getting in.
Safeties: You will likely be admitted to these programs if you present your case credibly.

 

How Competitive Are You?
How do you determine which schools fall under these three categories, when a safety school for one applicant could be a reasonable reach for another, and vice versa? Looking at the rankings, class profiles, and application requirements will help you figure out which schools are harder or easier to get into, but none of this matters without comparing those stats and figures to your own personal admissions profile.

 

Let’s take a look at the following categories:

1. GPA & GMAT
How do your stats stack up when compared to the mid 75-80% of students at a given program?

To gain a status of a safety school, you’ll want to fall into the top one-third of this range; to be considered an on-par, you’ll need to make it to the mid-third; and if you’re in the lower third, then this school would be a reasonable reach for you.

 

2.Work experience
The next factor to consider is your work experience, both qualitatively and quantitatively.

Top tier programs – perhaps those on your reasonable reach list – will want high quality work experience where you can show advancement, impact, and leadership relative to your accomplished peers, regardless of your function, industry, or organization. These top programs will turn down applicants with near-perfect stats if they lack the requisite professional accomplishment.

The number of years you’ve worked is also important: having fewer than three or more than eight years of work experience may make you less competitive.

 

3.Your demographic
This is another factor to consider when weighing your competitiveness – your demographic group. This includes the global region/country you’re from, your cultural background, and your industry.

Some demographic profiles are overrepresented in b-school admissions (for example, Indian IT males) so if you have this “label,” you’ll need to sharpen your competitive edge (even more than usual) if you’re applying to reasonable reaches. But that doesn’t mean that applicants from underrepresented groups are a shoo-in at these schools – if you can’t handle the program, you won’t get in, no matter how unique your personal background is.

 

A Fourth Category: Out-of-Reaches
There’s a fourth category that we didn’t mention: the out-of-reaches. While it’s true that you should never say never and that nothing is impossible, we generally believe that there are much better ways of spending your time, money, and energy than applying to these super-hard-to-get-into (for you) programs.

 

Bottom Line
As you research and visit MBA programs, determine how qualified and competitive you are for them. Then draft a list of reasonable reaches, on-parts, and safeties to apply to. And then apply with confidence knowing that you’re applying to an amazing mix of schools for you.

Having trouble choosing the best programs for you? Grab your free copy of Best MBA Programs: A Guide to Selecting the Right One, an exclusive Accepted admissions guide that will help you hand-pick the ideal MBA programs for YOU.

 

 

The post The 3 Types of B-Schools You Should Apply To appeared first on Magoosh GMAT Blog.
This Blog post was imported into the forum automatically. We hope you found it helpful. Please use the Kudos button if you did, or please PM/DM me if you found it disruptive and I will take care of it. -BB
Magoosh GMAT Instructor
Joined: 28 Dec 2011
Posts: 4490
Own Kudos [?]: 28872 [0]
Given Kudos: 130
GMAT Updates – 2016 [#permalink]
Expert Reply
FROM Magoosh Blog: GMAT Updates – 2016
On Thursday, March 10, 2016, just a week before Mike’s favorite ethic holiday, GMAC, the folks who create the GMAT, hosted a webinar for test prep professionals, folks such as I, on GMAT updates and I listened in. Here are some of the discoveries I made.

 

Canceling and Reinstating Scores
Back in the old days (five years ago), at the end of one’s GMAT, one had to decide there and then whether to cancel the score, before even seeing one’s score. One was shooting the in the dark. That was the only chance to cancel, and one had a 60-day window to reverse the cancelation: one reversed this by calling GMAC on the phone. Finally, for every canceled score, a “C” would appear on the score report, notifying adcom that the candidate had sat for a GMAT and had canceled it.

A great deal has changed. In the past year, the GMAT allowed student to preview their GMAT score right at the end of the test, before deciding to cancel. Since then, student have not be shooting in the dark.

In a new blog, GMAC recently has announced a number of changes.

a) the “C” will no longer appear on score reports; the score reports will contain absolutely no evidence of a cancelled GMAT!!

b) students have up to 72 hours after the GMAT to cancel the score (for $25): it doesn’t have to be made sitting in the testing center

c) you can reinstate a canceled score up to 4 years and 11 months after sitting for the GMAT (for $50), and you can do this by phone or online.

It’s wonderful that GMAC offers all these options. I would say: that $25 to cancel and $50 to reinstate is essentially a $75 penalty for not being well prepared. I would say: have a realistic plan. Know what scores are acceptable and what scores would merit a cancelation before you ever walk into the test center: make sure you discuss this plan with friends, with teachers, with admission counsellors, before you ever walk into the test. Be well appraised of the possibility of scoring lower on the real GMAT than you have on practice tests. Be intentional and conscientious and thoughtful about your approach to the GMAT, to business school, to your career, and to your life: this alone, when done consistently, will put you way ahead of others.

 

Integrated Reasoning
The folks at GMAC are quite fond of the IR section of the GMAT. A couple of years ago, many GMATs sent to B-school adcom were still pre-IR GMATs from early 2012. At this point, over 99% of GMATs sent to B-school have an IR section. Now that the IR is a part of virtually every GMAT the folks on adcom see, they are in a position to use it more as an assessment tool. Data suggests that the IR is often used in tie-breaking situations between two close candidates. It is not unreasonable to expect to see the importance of the IR section continuing to increase over time.

 

New Products
In 2016, look for the release of the following new products from GMAC

1) Recently, they released Exam Pack 2. Of course, students still can download the original GMAT Prep software for free, and this contains two practice GMATs. In the past, students could buy Exam Pack 1 and get two more full-length practice GMATs. Now, if one buys both Exam Packs, one can have a grand total of six full-length practice GMATs full of official questions.

NOTE: I highly recommend that all students studying for the GMAT take at least a couple full-length GMATs of official questions before they take the GMAT. Having said that, if you plan to take a lot of practice tests, use other practice tests earlier in your studying, and save the official tests toward the final days of your preparation.

2) The newest edition of the GMAT OG, the OG 2017, will be released in May 2016. Should you rush out and get this newest edition? Well, if you don’t own a GMAT OG yet, and you need to buy one, you might as well buy the most recent edition. If you already own the current OG, or even an OG from one or two editions ago, don’t panic: that’s absolutely fine to prepare you for the GMAT. If you absolutely run out of questions in your current edition of the GMAT, and need more official questions, then go ahead and by the newest edition when it comes out. If you have to buy the OG because you don’t own one yet or have exhausted your current guide, then those are truly valid reasons to make the purchase. Don’t buy the new one simply because it is new. Don’t automatically assume that the value of the current OG will drop to sheer worthlessness when the new one comes out: that’s patently absurd!! In general, I would say it would be extremely healthy to break yourself of any reflexive lurch toward the “next best thing.’

3) GMAC already sells Question Pack 1. In the second half of 2016, look for the release of Question Pack 2.

 

The Test Itself
At the moment, no new changes have been announced for the GMAT itself. The last big change made to the test was the addition of the Integrated Reasoning in June, 2012. The folks at GMAC are exceptionally proud of the quality and statistical validity of the test they produce, as well they should be!! There are no plans to make any major changes to the content of the test.

Having said that, on the webinar, they spoke of a pilot program that possibly could get expanded to all test takers at some point in the future. In this experimental program, students get a choice about the order of the sections. One still has to do all the sections (AWA, IR, Quantitative, and Verbal), but rather than have those four sections locked in that order, one could choose the order. It’s unclear how much choice one would have—whether, for example, one could opt for any of the 4! = 24 permutations of those four sections, or whether the AWA and IR, the two 30 minute sections, would always be “bundled.” Nevertheless, this could be a huge help to some students. If I were to take a GMAT under these conditions, I would want to take the Verbal section first because that requires my mind to be sharper—when I am half-fried I can still do a lot of math!

There is no timeline for this feature, and there’s no guarantee even that, on the basis of these trials, GMAC will decide to implement this feature. It’s in the testing phase at the moment and anything could happen from there.

 

Summary
If you have any experiences with GMAC’s new products or policies, please let us know in the comment sections. Of course, if you want serious help in preparing for the GMAT, Magoosh will provide as much help as any program that costs anywhere from 10x to 50x as much as Magoosh’s price. You can spend more money if you like—that is entirely your choice—but you won’t find anything significantly better than Magoosh, no matter how much money you spend.

 

The post GMAT Updates – 2016 appeared first on Magoosh GMAT Blog.
This Blog post was imported into the forum automatically. We hope you found it helpful. Please use the Kudos button if you did, or please PM/DM me if you found it disruptive and I will take care of it. -BB
Magoosh GMAT Instructor
Joined: 28 Dec 2011
Posts: 4490
Own Kudos [?]: 28872 [0]
Given Kudos: 130
GMAT Tuesday: Must Know Idioms #15 [#permalink]
Expert Reply
FROM Magoosh Blog: GMAT Tuesday: Must Know Idioms #15
Another Tuesday filled with idioms. Every where you turn, there are new idioms to learn and take in. And if you put in the time to learn them, you’ll breeze through some of the sentence correction questions on the GMAT. 😀

This week we look at fascinated, compare, replace, and account idiomatic phrases.



 

Check out this week’s board:


 

The post GMAT Tuesday: Must Know Idioms #15 appeared first on Magoosh GMAT Blog.
This Blog post was imported into the forum automatically. We hope you found it helpful. Please use the Kudos button if you did, or please PM/DM me if you found it disruptive and I will take care of it. -BB
Magoosh GMAT Instructor
Joined: 28 Dec 2011
Posts: 4490
Own Kudos [?]: 28872 [0]
Given Kudos: 130
Admission Tips: Countering a Low Verbal Score [#permalink]
Expert Reply
FROM Magoosh Blog: Admission Tips: Countering a Low Verbal Score
MBA applicants know too well the pain of receiving a lousy verbal GMAT score. It can feel like the end of your career before you’ve even started, but that doesn’t have to be the case. Since b-school students are expected to have excellent communications skills, it’s not easy, but it is also not impossible to get accepted even with a low verbal GMAT score.

Here are three simple ways to show the adcoms that your scores are not showing the full picture, and that you’re still a competitive candidate worthy of that golden acceptance.

 

1. Counter low scores with high scores
The easiest way to prove a single score wrong is by having other grades that tell a different story. Consider taking additional classes that will highlight your strong verbal/communication skills. Take communications classes at a local college, participate in a local debate club, take a public speaking course – and then, of course, get straight A’s or rave reviews in all of them! – to balance out the poor verbal score.

This is a good tip for anyone who scored low on their GMAT, but is even more important if you already have poor English grades on your transcript.

 

2. Wow them with your winning essay-writing abilities
Another proven way to counter a poor verbal GMAT score is to illustrate that you can actually write eloquently and persuasively. Construct a flawless application essay, complete with examples, details, and stories that will adequately convey how well you handle your communications skills. Bringing Including real life examples of your abilities will speak much more strongly for your skills than just another well-written essay.

 

3. Get others to put in a good word
Speak to each one of the people you’ve asked to write a recommendation for you, and ask them to highlight some of your stronger verbal or communications skills. Once again, bringing in actual examples of instances or areas that you have developed and utilized these skills will help to build the argument even more.

The GMAT is just one element in a grander picture, so make sure the adcoms are seeing the wider view of your capabilities by using some of these useful tips when you apply to b-school. For more advice, view our on-demand webinar, Get Accepted to B-School with Low Stats.

The post Admission Tips: Countering a Low Verbal Score appeared first on Magoosh GMAT Blog.
This Blog post was imported into the forum automatically. We hope you found it helpful. Please use the Kudos button if you did, or please PM/DM me if you found it disruptive and I will take care of it. -BB
Magoosh GMAT Instructor
Joined: 28 Dec 2011
Posts: 4490
Own Kudos [?]: 28872 [0]
Given Kudos: 130
GMAT Tuesday: Must Know Idioms #16 [#permalink]
Expert Reply
FROM Magoosh Blog: GMAT Tuesday: Must Know Idioms #16
Dive into another week of Idioms! In this weeks video, I cover idiomatic phrases involving hold, view, know, and hope. 😀

Be excellent to the universe!



Here’s this week’s board!


 

The post GMAT Tuesday: Must Know Idioms #16 appeared first on Magoosh GMAT Blog.
This Blog post was imported into the forum automatically. We hope you found it helpful. Please use the Kudos button if you did, or please PM/DM me if you found it disruptive and I will take care of it. -BB
Magoosh GMAT Instructor
Joined: 28 Dec 2011
Posts: 4490
Own Kudos [?]: 28872 [0]
Given Kudos: 130
Intro to GMAT Word Problems, Part 1: Translating from Word to Math [#permalink]
Expert Reply
FROM Magoosh Blog: Intro to GMAT Word Problems, Part 1: Translating from Word to Math
First, a four GMAT practice word problems.

1) Seven more than a number is 2 more than four times the number.  What is the number?

(A) 1

(B) 2/3

(C) 3/5

(D) 4/7

(E) 5/3

 

2) If $40,000 less than John’s salary is $5,000 more than 25% more than half his salary, then what is John’s salary?

(A) $80,000

(B) $100,000

(C) $120,000

(D) $135,000

(E) $140,000

 

3) Twice a number is 3 times the square of the number less than one.  If the number is positive, what is the value of the number?

(A) 1

(B) 1/2

(C) 1/3

(D) 2/3

(E) 3/2

 

4) The original price of an item is discounted 20%. A customer buys the item at this discounted price using a $20-off coupon.  There is no tax on the item, and this was the only item the customer bought.  If the customer paid $1.90 more than half the original price of the item, what was the original price of the item?

(A) $61

(B) $65

(C) $67.40

(D) $70

(E) $73

 

GMAT word problems
It’s one thing to understand algebra in the abstract, and quite another to think about where the rubber meets the road.  The reason human beings created algebra was to solve problems about real world situations, and the GMAT loves asking math problems about numbers and about real world situations, a.k.a. word problems!  Even folks who can do algebra in the abstract sometimes find word problems challenging.  In this blog and the next, we present a rough-and-ready guide to what you need about word problems.

 

Translating from Words to Math
Suppose we have the following sentence in a word problem:

“Three-fifths of x is 14 less than twice y squared.”

How do we change words to math?  Here’s a quick guide

1) the verb “is/are” is the equivalent of an equal sign; the equal sign in an equation is, in terms of “mathematical grammar,” the equivalent of a verb in a sentence.  Every sentence has a verb and every equation has an equal sign.

2) The word “of” means multiply (often used with fractions and percents).  Ex. “26% of x” means (0.26x)

3) The words “more than” or “greater than” mean addition.  Ex. “5 greater than x” means (x + 5) and “7 more than y” means (y + 7)

4) The words “less than” means subtraction.  Ex. “8 less than Q” means (Q – 8).  Notice that the first element is always subtracted: in other words, “J less than K” means (K – J).

With that in mind, let’s go back to the sentence from the hypothetical problem above.

“three fifths of x” means [(3/5)*x]

“is” marks the location of the equal sign

“twice y squared” means 2(y^2)

“14 less than twice y squared” means 2(y^2) – 14

Altogether, the equation we get is:



Using this strategy, it’s straightforward to translate from a verbal statement about numbers to an equation.

 

Other strategies
When all the answer choices are numerical, one further strategy we have at our disposal is backsolving.  Using this strategy, we can pick one answer, plug it into the problem, and see whether it works.  If this choice is too big or too small, it guides us in what other answer choices to eliminate.  Typically, we would start with answer choice (C), but if another answer choice is a particularly convenient choice, then we would start there.

 

Summary
If the strategies discussed here gave you any insights, you may want to give the problems above another look before turning to the solutions below.  Look for the second article on Assigning Variables in Word Problems.



 

Explanations to Practice Problems
1) Translate this one step at a time.  Let N be the number we seek.

“seven more than a number” = N + 7

The “is” is the equal sign.

“two more than four times the number” = 4N + 2



Answer = (E)

2) We will say that S is John’s salary.  “$40,000 less than John’s salary” is (S – 40000).  The second part is tricky: “25% more than half his salary” is (1.25*(1/2)*S), so “$5,000 more than 25% more than half his salary” would be that plus 5000.  We can write the whole first part of the prompt sentence as



Multiply both sides by 8 (we don’t have to perform the numerical multiplication yet)

8S – 8*45000 = 5S

3S = 8*45000

S = 8*15000 = 4*30000 = 120,000

Notice the use of the doubling and halving trick to perform the multiplication in the last line.  John’s salary must be $120,000.

Answer = (C)

3) Call the number x.  Of course, “twice the number” equals 2x.  The part after the word “is” can be tricky.  Remember that the information of the form “J less than K” takes the mathematical form (K – J), in which the first part is the part that’s subtracted.  What we have here is “3 times the square of the number less than one.”  That would be one minus 3 times x squared. Now we can translate that entire first sentence of the prompt into math:



This is a quadratic.  We need to get all the terms on one side, equal to zero, and then factor.



Because the prompt tells us that the number must be positive, we can reject the negative root.  The number must be +1/3.

Answer = (C)

4)  We will show two solutions for this: (i) backsolving, and (ii) the full algebraic solution.

For the backsolving solution, notice that (C) is an ugly number.  (B) and (D) are nicer numbers.  Let’s start with (D).

Original price = $70

10% of price = $7, so

20% of price = $14.

After discount, the price is 70 – 14 = $56.  The customer then uses a $20-off coupon, so this customer pays $36.

How does this price compare to half the original price?  Well, half the original was $35, so the customer paid exactly $1 more than half the original price.

First of all, we know that answer choice (D) does not work.  We know we need a bigger difference, so we need a bigger price.  The only price bigger is (E).  This must be the answer.

Answer = (E)

Now, a full algebra solution.  Let the original price be P.  Then, 20% off would be 0.8*P.  Then, if we subtract $20, that’s (0.8*P – 20).  That is the price the customer paid, which equals “$1.90 more than half the original price,” or (0.5P + 1.9).  We will set these equal.

08*P – 20 = 0.5*P + 1.9

0.3*P = 21.9

3P = 219

P = 73

The original price was $73.  Answer = (E)

 

The post Intro to GMAT Word Problems, Part 1: Translating from Word to Math appeared first on Magoosh GMAT Blog.
This Blog post was imported into the forum automatically. We hope you found it helpful. Please use the Kudos button if you did, or please PM/DM me if you found it disruptive and I will take care of it. -BB
User avatar
Manager
Manager
Joined: 18 Feb 2016
Posts: 87
Own Kudos [?]: 63 [0]
Given Kudos: 94
Send PM
Magoosh Blog [#permalink]
I think they are good.... a friend of mine told me that he took a course with them and recommend them
Magoosh GMAT Instructor
Joined: 28 Dec 2011
Posts: 4490
Own Kudos [?]: 28872 [0]
Given Kudos: 130
Past Perfect on GMAT Sentence Correction [#permalink]
Expert Reply
FROM Magoosh Blog: Past Perfect on GMAT Sentence Correction
When we speak of one past event that happened before another past event, one way to denote the earlier event is by use of the past perfect tense.  When should we use the past perfect, and when is it not required?  This is a tricky issue.   First, four practice sentence correction questions.

 

1) James Joyce wrote the novel Ulysses through the teens and published it in 1922, although in 1906, when he was finishing the short story collection Dubliners, he had considered the addition of another story about the canvasser Bloom, who was Jewish and who later was the title Ulysses character.

(A) he had considered the addition of another story about the canvasser Bloom, who was Jewish and who later was the title Ulysses character.

(B) he had been considering that he add an additional extra story, the Jewish canvasser Bloom, and to later make him the Ulysses‘ title character.

(C) considering the inclusion of the story of Bloom, the Jewish canvasser who would become the Ulysses title character later

(D) he considered including another story about the Jewish canvasser Bloom, who later would be the title character of the Ulysses

(E) he considered adding the Jewish canvasser Bloom’s story, and Bloom later would have become the Ulysses title character

 

2) Long before Thomas Edison made a long-lasting and commercially viable incandescent lightbulb, the British chemist Sir Humphry Davy in 1802 created the first prototype of the lightbulb, that was too bright and burned out too quickly.

(A) in 1802 created the first prototype of the lightbulb, that was

(B) made the first prototype in 1802, but it was

(C) had created in 1802 the first prototype of the lightbulb, although this would be

(D) had created the 1802 prototype, the first one, which had been

(E) in 1802 had made the first prototype, but this would be

 

3)  In 1871, Charles Darwin published The Descent of Man, and before that, Gregor Mendel already had discovered the principles of genetics, using his famous pea plant experiments, and ultimately they would explain and justify Darwin’s conclusions.

(A) In 1871, Charles Darwin published The Descent of Man, and before that, Gregor Mendel already had discovered the principles of genetics, using his famous pea plant experiments, and ultimately they would explain and justify Darwin’s conclusions

(B) By the time Charles Darwin published The Descent of Man in 1871, Gregor Mendel already had discovered, during his famous pea plant experiments, the genetic principles that ultimately would explain and justify Darwin’s conclusions

(C) Gregor Mendel already discovered the principles of genetics during his famous pea plant experiments, and although later Charles Darwin published The Descent of Man in 1871, these principles ultimately would explain and justify the conclusions of Darwin.

(D) With Charles Darwin publishing The Descent of Man in 1871, Gregor Mendel discovered already before this the principles of genetics during his famous pea plant experiments, principles that ultimately would explain and justify the conclusions of Darwin

(E) Before Charles Darwin publishing The Descent of Man in 1871, already Gregor Mendel had conducted the famous pea plants experiments and had discovered the principles of genetics, but these principles ultimately would explain and justify Darwin’s conclusions

 

4) Had the quantitative skills of Boustrophedon’s CEO been on par with his extraordinary intuition in negotiations, he would not have needed such a brilliant expert in finance as his CFO.

(A) Had the quantitative skills of Boustrophedon’s CEO been on par with his extraordinary intuition in negotiations

(B) If the Boustrophedon’s CEO’s quantitative skills equaled his extraordinary negotiation intuition

(C) In the case that the quantitative skills of Boustrophedon’s CEO were comparable to his extraordinary skill of using intuition in negotiations

(D) If Boustrophedon’s CEO’s skills for quantitative material were on par to his extraordinary intuition in negotiations

(E) If the quantitative skills of Boustrophedon’s CEO were a comparable ability for his extraordinary intuition in negotiations

Complete explanations will follow this article.

The past perfect tense
First of all, here are two previous posts:

GMAT Verb Tense: The Perfect Tense

GMAT Verb: The Perfect Tense?

As you may remember, or as may have been reminded in reading one of those two blogs, the pat perfect tense indicates an action that precedes another past action.  It is one way to indicate a sequence of events, all of which are in the past.  For example

5) George Washington was a skilled military commander at the outset of the American Revolution because he had gained extensive battlefield experience in the French and Indian War.

In this example sentence, the verb “had gained” is in the past perfect, to make clear that it indicates an action before the time of the verb “was.”  If you know American history, you might have known that the French and Indian War ended about a decade before the American Revolutionary War began.  That knowledge is outside this sentence, and of course, such outside knowledge is not required to understand GMAT SC questions, so if sentence #5 were a sentence in a GMAT SC question, the use of the past perfect would tell us all we need to know about which action came first.

Necessary or redundant?
The past perfect tense is definitely one way to indicate that one past action was before another, but it’s not the only way to do so.   Consider this sentence,

6a) Richard Nixon was elected US President in 1968, but as the incumbent Vice President he had run in 1960 and had been defeated by John F. Kennedy

Notice that this sentence gives us two different ways to know that the events in the second half of the sentence were earlier in time.  One is use of the past perfect, and the other is simply the dates given.  We know that 1960 happened before 1968: unlike sentence #5, this sentence supplies us with the exact historical information so we know the years when events happened.  We don’t need outside knowledge, as we would have with #5.

In a way, this is a kind of redundancy, because the information about which event was earlier is given in two different ways.  It’s not as glaring as other forms of redundancy, but it’s certainly suspicious to the GMAT.  The GMAT will often consider use of the past perfect tense incorrect if there are other clear indicators of time sequence in the sentence.  In all likelihood, the GMAT would prefer this version:

6b) Richard Nixon was elected US President in 1968, but as the incumbent Vice President he ran in 1960 and was defeated by John F. Kennedy.

This version, with the verbs of the second half in the simple past tense, is no longer redundant: only the dates given indicate the sequence.  As a general rule, if dates or other indications in the sentence clearly let us know the sequence of events, then the use of the past perfect is certainly no necessary and may be considered redundant and incorrect.

Conditionals
The past perfect tense involves the use of the auxiliary verb “had” before the past participle form of the main verb.   In formal writing, this opens up an alternative possible structure for conditionals.

Typically, conditional statements start with the  word “if.”  If the verb employs the auxiliary verb “had,” “should,” or “were,” we can construct a conditional statement by omitting the word “if” and putting the auxiliary verb before the subject.  Consider this standard conditional sentence.

7a) If I had known beforehand about the full cost of the program, I never would have joined.

That is a 100% grammatical correct sentence, but it is a bit prosaic.  It is completely factual and lacking in elegance.  By contrast, consider this sophisticated rewrite.

7b) Had I known beforehand about the full cost of the program, I never would have joined.

This conveys precisely the same factual information, but unlike it’s prosaic partner #7a, this version has an air of sophistication.   Similarly,

8) Should he win the first round, he will find the competition in the second round considerably more daunting. 

9) Were I President of the US, I would make “My Country, ‘Tis of Thee” the national anthem. 

These are further examples of conditional statements in which the word “if” has been dropped and the auxiliary verb moved to the beginning of the sentence.  Again, this is a very sophisticated structure, typical of high quality writing.

Summary
If the ideas discussed here gave you any insights into the practice questions at the top, you may want to look at them again before reading the explanations below.  As you read GMAT RC practice and sophisticated reading unrelated to the GMAT, notice when the past perfect is and isn’t used, and notice the sophisticated conditional structure discussed in the last section.



 

Practice Question Explanations
1) A question about Mike’s favorite author, James Joyce, and his favorite novel, Ulysses.  The main character is Leopold Bloom, and the book is celebrated on Bloomsday.

Because we have years, the past perfect tense is not necessary.

(A) This is grammatically correct but long-winded and a bit awkward.  Also, the phrasing “title Ulysses character” is very awkward and unnatural.  This is incorrect.

(B) This is a a disaster.  The progressive tense “considering” is unnecessary.  Having a “that” clause following this is a wordy and awkward way to convey this information. The big mistake: this version has a failure in parallelism: “that he add . . . and to later make.”  Finally, just a detail, but that last infinite is a split infinitive, a gaff that would not appear on the correct answer of a GMAT SC problem.  This is incorrect.

(C) This commits the famous missing verb mistake.  The “although” clause should have a full subject + verb after the “when” clause, but all we get here is a participle.  This is a gigantic grammatical failure.  This is incorrect.

(D) Elegant and mistake free.  This is promising.

(E) The phrasing “the Jewish canvasser Bloom’s story” is a little awkward, and the hypothetical phrasing in the second half changes the meaning, seeming to suggest that Bloom did not become the title character of the Ulysses.  This meaning is different from that of the prompt.   This is incorrect.

The only possible answer is (D).

 

2) A sentence that mentions the inventor Thomas Edison (1847 – 1931) and the scientist Sir Humphry Davy (1778 – 1829).

(A) This has no major mistake but it is awkward.  It is awkward that the sentence uses “that” for a non-restrictive modifier.  Also, it’s not necessary to repeat the word “lightbulb,” since this is the topic of the sentence.  This is incorrect.

(B) This is elegant and promising.

(C) The past perfect tense is not necessary and somewhat redundant, because time sequence is already double indicated by the “long before” phrase and the years.  The hypothetical tone at the end is not appropriate.  This is incorrect.

(D) Again, the past perfect is not necessary.  Also, the phrasing “the 1802 prototype, the first one” is quite awkward and unclear.  This is incorrect.

(E) Again, the past perfect is not necessary.  Again, the hypothetical tone at the end is not appropriate.  This is incorrect.

The only possible answer is (B).

 

3) A sentence about two great scientists, Charles Darwin (1809 – 1882) and Gregor Mendel (1822 – 1884).  The whole sentence is underlined, so we have to look at each choice separately.

(A) The “before that” and use of the past perfect could be considered redundant.  Also, the pronoun “they” in the last clause is ambiguous in its referent.  This is incorrect.

(B) This correctly uses the past perfect tense as the only indication of time sequence.  This is elegant and mistake-free—a promising choice.

(C) This is grammatically correct but awkward.  The “already” in the first clause would seem to establish a connection with other actions, but this expectation is not met.  Also, the huge logical shift created by “although” makes the logical relationship of the second clause to the first clause quite unclear.   This is incorrect.

(D) This choice uses the “with” + [noun] + [participle] structure, and uses it to encapsulate action by an actor other than the subject of the sentence.  That’s too much action inside a prepositional phrase and Mendel did not do what he did “with Darwin.”  Also, the phrase “already before this” is extremely awkward.  This is incorrect.

(E) This choice commits the famous missing verb mistake in the opening subordinate clause.  After the subordinate conjunction “before,” we need a full noun + verb structure, and rather than a full verb, we get only a participle.   The “but” before the final clause is illogical.  This is incorrect.

The only possible answer is (B).

 

4) (A) This first option employs the sophisticated conditional structure discussed in the section on “Conditionals” in this blog.   This is grammatically and idiomatically correct, and it makes use of an elegant structure.  This is a promising choice.

(B) This is awkward.  First of all, the phrases “the Boustrophedon’s CEO’s quantitative skills” and “his extraordinary negotiation intuition” are both awkward insofar as they pile too many nouns together at once.  Also, the word “equaled” is too strong: it suggests that two things are are entirely equivalent, such that one could substitute for the other, and this reading changes the meaning from the prompt.  This is incorrect.

(C) This is grammatically correct but very long, wordy, and awkward.  At every instance, this option chooses an especially wordy way to phrase something, so the whole thing is a rambling disaster.  This is incorrect.

(D) The double possessive, “Boustrophedon’s CEO’s skills,” is somewhat awkward: this is a structure unlikely to appear as part of a correct answer on an official GMAT SC question.   Also, the phrase “on par to” is idiomatically incorrect; the correct idiom is “on par with.”  This is incorrect.

(E) The phrase “were a comparable ability for” is not idiomatically correct to convey a comparison.  This phrase could be correct if the object of “for” were something other than the second term in the comparison.  This is incorrect.

The only possible answer is (A).

 

 

The post Past Perfect on GMAT Sentence Correction appeared first on Magoosh GMAT Blog.
This Blog post was imported into the forum automatically. We hope you found it helpful. Please use the Kudos button if you did, or please PM/DM me if you found it disruptive and I will take care of it. -BB
Magoosh GMAT Instructor
Joined: 28 Dec 2011
Posts: 4490
Own Kudos [?]: 28872 [0]
Given Kudos: 130
Past Perfect on GMAT Sentence Correction [#permalink]
Expert Reply
FROM Magoosh Blog: Past Perfect on GMAT Sentence Correction
When we speak of one past event that happened before another past event, one way to denote the earlier event is by use of the past perfect tense.  When should we use the past perfect, and when is it not required?  This is a tricky issue.   First, four practice sentence correction questions.

 

1) James Joyce wrote the novel Ulysses through the teens and published it in 1922, although in 1906, when he was finishing the short story collection Dubliners, he had considered the addition of another story about the canvasser Bloom, who was Jewish and who later was the title Ulysses character.

(A) he had considered the addition of another story about the canvasser Bloom, who was Jewish and who later was the title Ulysses character.

(B) he had been considering that he add an additional extra story, the Jewish canvasser Bloom, and to later make him the Ulysses‘ title character.

(C) considering the inclusion of the story of Bloom, the Jewish canvasser who would become the Ulysses title character later

(D) he considered including another story about the Jewish canvasser Bloom, who later would be the title character of the Ulysses

(E) he considered adding the Jewish canvasser Bloom’s story, and Bloom later would have become the Ulysses title character

 

2) Long before Thomas Edison made a long-lasting and commercially viable incandescent lightbulb, the British chemist Sir Humphry Davy in 1802 created the first prototype of the lightbulb, that was too bright and burned out too quickly.

(A) in 1802 created the first prototype of the lightbulb, that was

(B) made the first prototype in 1802, but it was

(C) had created in 1802 the first prototype of the lightbulb, although this would be

(D) had created the 1802 prototype, the first one, which had been

(E) in 1802 had made the first prototype, but this would be

 

3)  In 1871, Charles Darwin published The Descent of Man, and before that, Gregor Mendel already had discovered the principles of genetics, using his famous pea plant experiments, and ultimately they would explain and justify Darwin’s conclusions.

(A) In 1871, Charles Darwin published The Descent of Man, and before that, Gregor Mendel already had discovered the principles of genetics, using his famous pea plant experiments, and ultimately they would explain and justify Darwin’s conclusions

(B) By the time Charles Darwin published The Descent of Man in 1871, Gregor Mendel already had discovered, during his famous pea plant experiments, the genetic principles that ultimately would explain and justify Darwin’s conclusions

(C) Gregor Mendel already discovered the principles of genetics during his famous pea plant experiments, and although later Charles Darwin published The Descent of Man in 1871, these principles ultimately would explain and justify the conclusions of Darwin.

(D) With Charles Darwin publishing The Descent of Man in 1871, Gregor Mendel discovered already before this the principles of genetics during his famous pea plant experiments, principles that ultimately would explain and justify the conclusions of Darwin

(E) Before Charles Darwin publishing The Descent of Man in 1871, already Gregor Mendel had conducted the famous pea plants experiments and had discovered the principles of genetics, but these principles ultimately would explain and justify Darwin’s conclusions

 

4) Had the quantitative skills of Boustrophedon’s CEO been on par with his extraordinary intuition in negotiations, he would not have needed such a brilliant expert in finance as his CFO.

(A) Had the quantitative skills of Boustrophedon’s CEO been on par with his extraordinary intuition in negotiations

(B) If the Boustrophedon’s CEO’s quantitative skills equaled his extraordinary negotiation intuition

(C) In the case that the quantitative skills of Boustrophedon’s CEO were comparable to his extraordinary skill of using intuition in negotiations

(D) If Boustrophedon’s CEO’s skills for quantitative material were on par to his extraordinary intuition in negotiations

(E) If the quantitative skills of Boustrophedon’s CEO were a comparable ability for his extraordinary intuition in negotiations

Complete explanations will follow this article.

The past perfect tense
First of all, here are two previous posts:

GMAT Verb Tense: The Perfect Tense

GMAT Verb: The Perfect Tense?

As you may remember, or as may have been reminded in reading one of those two blogs, the pat perfect tense indicates an action that precedes another past action.  It is one way to indicate a sequence of events, all of which are in the past.  For example

5) George Washington was a skilled military commander at the outset of the American Revolution because he had gained extensive battlefield experience in the French and Indian War.

In this example sentence, the verb “had gained” is in the past perfect, to make clear that it indicates an action before the time of the verb “was.”  If you know American history, you might have known that the French and Indian War ended about a decade before the American Revolutionary War began.  That knowledge is outside this sentence, and of course, such outside knowledge is not required to understand GMAT SC questions, so if sentence #5 were a sentence in a GMAT SC question, the use of the past perfect would tell us all we need to know about which action came first.

Necessary or redundant?
The past perfect tense is definitely one way to indicate that one past action was before another, but it’s not the only way to do so.   Consider this sentence,

6a) Richard Nixon was elected US President in 1968, but as the incumbent Vice President he had run in 1960 and had been defeated by John F. Kennedy

Notice that this sentence gives us two different ways to know that the events in the second half of the sentence were earlier in time.  One is use of the past perfect, and the other is simply the dates given.  We know that 1960 happened before 1968: unlike sentence #5, this sentence supplies us with the exact historical information so we know the years when events happened.  We don’t need outside knowledge, as we would have with #5.

In a way, this is a kind of redundancy, because the information about which event was earlier is given in two different ways.  It’s not as glaring as other forms of redundancy, but it’s certainly suspicious to the GMAT.  The GMAT will often consider use of the past perfect tense incorrect if there are other clear indicators of time sequence in the sentence.  In all likelihood, the GMAT would prefer this version:

6b) Richard Nixon was elected US President in 1968, but as the incumbent Vice President he ran in 1960 and was defeated by John F. Kennedy.

This version, with the verbs of the second half in the simple past tense, is no longer redundant: only the dates given indicate the sequence.  As a general rule, if dates or other indications in the sentence clearly let us know the sequence of events, then the use of the past perfect is certainly no necessary and may be considered redundant and incorrect.

Conditionals
The past perfect tense involves the use of the auxiliary verb “had” before the past participle form of the main verb.   In formal writing, this opens up an alternative possible structure for conditionals.

Typically, conditional statements start with the  word “if.”  If the verb employs the auxiliary verb “had,” “should,” or “were,” we can construct a conditional statement by omitting the word “if” and putting the auxiliary verb before the subject.  Consider this standard conditional sentence.

7a) If I had known beforehand about the full cost of the program, I never would have joined.

That is a 100% grammatical correct sentence, but it is a bit prosaic.  It is completely factual and lacking in elegance.  By contrast, consider this sophisticated rewrite.

7b) Had I known beforehand about the full cost of the program, I never would have joined.

This conveys precisely the same factual information, but unlike it’s prosaic partner #7a, this version has an air of sophistication.   Similarly,

8) Should he win the first round, he will find the competition in the second round considerably more daunting. 

9) Were I President of the US, I would make “My Country, ‘Tis of Thee” the national anthem. 

These are further examples of conditional statements in which the word “if” has been dropped and the auxiliary verb moved to the beginning of the sentence.  Again, this is a very sophisticated structure, typical of high quality writing.

Summary
If the ideas discussed here gave you any insights into the practice questions at the top, you may want to look at them again before reading the explanations below.  As you read GMAT RC practice and sophisticated reading unrelated to the GMAT, notice when the past perfect is and isn’t used, and notice the sophisticated conditional structure discussed in the last section.



 

Practice Question Explanations
1) A question about Mike’s favorite author, James Joyce, and his favorite novel, Ulysses.  The main character is Leopold Bloom, and the book is celebrated on Bloomsday.

Because we have years, the past perfect tense is not necessary.

(A) This is grammatically correct but long-winded and a bit awkward.  Also, the phrasing “title Ulysses character” is very awkward and unnatural.  This is incorrect.

(B) This is a a disaster.  The progressive tense “considering” is unnecessary.  Having a “that” clause following this is a wordy and awkward way to convey this information. The big mistake: this version has a failure in parallelism: “that he add . . . and to later make.”  Finally, just a detail, but that last infinite is a split infinitive, a gaff that would not appear on the correct answer of a GMAT SC problem.  This is incorrect.

(C) This commits the famous missing verb mistake.  The “although” clause should have a full subject + verb after the “when” clause, but all we get here is a participle.  This is a gigantic grammatical failure.  This is incorrect.

(D) Elegant and mistake free.  This is promising.

(E) The phrasing “the Jewish canvasser Bloom’s story” is a little awkward, and the hypothetical phrasing in the second half changes the meaning, seeming to suggest that Bloom did not become the title character of the Ulysses.  This meaning is different from that of the prompt.   This is incorrect.

The only possible answer is (D).

 

2) A sentence that mentions the inventor Thomas Edison (1847 – 1931) and the scientist Sir Humphry Davy (1778 – 1829).

(A) This has no major mistake but it is awkward.  It is awkward that the sentence uses “that” for a non-restrictive modifier.  Also, it’s not necessary to repeat the word “lightbulb,” since this is the topic of the sentence.  This is incorrect.

(B) This is elegant and promising.

(C) The past perfect tense is not necessary and somewhat redundant, because time sequence is already double indicated by the “long before” phrase and the years.  The hypothetical tone at the end is not appropriate.  This is incorrect.

(D) Again, the past perfect is not necessary.  Also, the phrasing “the 1802 prototype, the first one” is quite awkward and unclear.  This is incorrect.

(E) Again, the past perfect is not necessary.  Again, the hypothetical tone at the end is not appropriate.  This is incorrect.

The only possible answer is (B).

 

3) A sentence about two great scientists, Charles Darwin (1809 – 1882) and Gregor Mendel (1822 – 1884).  The whole sentence is underlined, so we have to look at each choice separately.

(A) The “before that” and use of the past perfect could be considered redundant.  Also, the pronoun “they” in the last clause is ambiguous in its referent.  This is incorrect.

(B) This correctly uses the past perfect tense as the only indication of time sequence.  This is elegant and mistake-free—a promising choice.

(C) This is grammatically correct but awkward.  The “already” in the first clause would seem to establish a connection with other actions, but this expectation is not met.  Also, the huge logical shift created by “although” makes the logical relationship of the second clause to the first clause quite unclear.   This is incorrect.

(D) This choice uses the “with” + [noun] + [participle] structure, and uses it to encapsulate action by an actor other than the subject of the sentence.  That’s too much action inside a prepositional phrase and Mendel did not do what he did “with Darwin.”  Also, the phrase “already before this” is extremely awkward.  This is incorrect.

(E) This choice commits the famous missing verb mistake in the opening subordinate clause.  After the subordinate conjunction “before,” we need a full noun + verb structure, and rather than a full verb, we get only a participle.   The “but” before the final clause is illogical.  This is incorrect.

The only possible answer is (B).

 

4) (A) This first option employs the sophisticated conditional structure discussed in the section on “Conditionals” in this blog.   This is grammatically and idiomatically correct, and it makes use of an elegant structure.  This is a promising choice.

(B) This is awkward.  First of all, the phrases “the Boustrophedon’s CEO’s quantitative skills” and “his extraordinary negotiation intuition” are both awkward insofar as they pile too many nouns together at once.  Also, the word “equaled” is too strong: it suggests that two things are are entirely equivalent, such that one could substitute for the other, and this reading changes the meaning from the prompt.  This is incorrect.

(C) This is grammatically correct but very long, wordy, and awkward.  At every instance, this option chooses an especially wordy way to phrase something, so the whole thing is a rambling disaster.  This is incorrect.

(D) The double possessive, “Boustrophedon’s CEO’s skills,” is somewhat awkward: this is a structure unlikely to appear as part of a correct answer on an official GMAT SC question.   Also, the phrase “on par to” is idiomatically incorrect; the correct idiom is “on par with.”  This is incorrect.

(E) The phrase “were a comparable ability for” is not idiomatically correct to convey a comparison.  This phrase could be correct if the object of “for” were something other than the second term in the comparison.  This is incorrect.

The only possible answer is (A).

 

 

The post Past Perfect on GMAT Sentence Correction appeared first on Magoosh GMAT Blog.
This Blog post was imported into the forum automatically. We hope you found it helpful. Please use the Kudos button if you did, or please PM/DM me if you found it disruptive and I will take care of it. -BB
GMAT Club Bot
Past Perfect on GMAT Sentence Correction [#permalink]
   1   2   3   4  ...  16